Вы находитесь на странице: 1из 72

HEMORRHAGE IN EARLY

ROBERT COLLIER: PREGNANCY


SUCCESS IS THE SUM OF CASE :
35 y/o G3P0 Pregnancy Uterine 12 weeks complains
of vaginal bleeding and hypogastric pain. Internal
SMALL EFFORTS,
examination revealed closed cervix ,uterus enlarged
to 3 months size, adnexae negative for mass or
REPEATED DAY IN AND DAY OUT tenderness.
WHAT IS THE DIAGNOSIS?
PATHOLOGIC OBSTETRICS WHAT ARE THE DIFFERENTIAL DIAGNOSIS?
BOARD REVIEW
Maria Carmen Hernandez-Quevedo, MD
Associate Professor B
Elizabeth Ahyong Reyes, M.D.
Associate Professor C

ABORTION INCIDENCE MATERNAL FACTORS


Pregnancy termination before the 20th week 80 % of spontaneous abortion occur within the 1.INFECTIONS-Chlamydia,polymicrobial infection
first 12 weeks from periodontal disease
Fetus born weighs less than 500 grams

50% anembryonic and 50% embryonic 2.MEDICAL DISORDERS-diabetes,thyroid


TYPES:
disorders,celiac disease,anorexia and bulimia,
EMBRYONIC cyanotic heart ,hypertension,inflammatory bowel
1.SPONTANEOUS- Euploid-(50%)normal chromosomes
disease,SLE
threatened,inevitable,incomplete,complet,missed. Aneuploid-(50%)abnormal chromosomes
Any of these with signs of infection-septic Trisomy(most common)
Monosomy X(second most common) 3.MEDICATIONS- IUD
2.RECURRENT-repetitive spontaneous abortion

Triploidy(molar)

4.CANCER-radiation(>5 rads) and methotrexate
3.INDUCED-surgical and medical termination of Tetraploidy

live fetus Structural anomaly

MATERNAL FACTORS MATERNAL FACTORS CLASSIFICATION OF ABORTION


5.SURGICAL PROCEDURES-bariatric surgery 9.IMMUNOLOGICAL-antiphospholipid antibody VAGINAL BLEEDING,HYPOGASTRIC PAIN
Give progesterone if done less than 10 weeks syndrome 1.THREATENED- cervix closed,uterus compatible
with age of gestation
AOG
2.IMMINENT-cervix open intact bag of waters.uterus
10.INHERITED THROMBOPHILIAS compatible with age of gestation
6.SOCIAL AND BEHAVIORAL FACTORS 3.INEVITABLE-cervix open,ruptured BOW,uterus
smaller than age of gestation
Alcohol,smoking ,caffeine(>5 cups/day) 11.UTERINE DEFECTS
4.INCOMPLETE-passage of meaty tissues,cervix
open,uterus smaller than age of gestation
7.OCCUPATIONAL/ENVIRONMENTAL 12.PATERNAL FACTORS 5.COMPLETE-passage of meaty tissues,cervix
FACTORS closed,uterus small
6.MISSED-cervix closed,uterus not compatible with
Arsenic,lead,formaldehyde,benzene,ethylene
age of gestation
oxide,DDT(dichlorodiphenylchloroethane)
ETIOLOGY OF RECURRENT
SEPTIC ABORTION RECURRENT MISCARRIAGE PREGNANCY LOSS
Grp.A strep/ Clostridium perfringens or sordelli Recurrentpregnancy loss,recurrent spontaneous Widely Accepted Causes
abortion,habitual abortion 1.Parental chromosomal abnormalities
Fever,leukocytosis,hemoconcentration,

Three or more consecutive spontaneous abortion 2.Antiphospholipid antibody syndrome-


hypotension,ARDS,kidney failure,DIC
autoimmune disorder(immunity against self)

Broad spectrum antibiotics Two or more failed clinical pregnancy confirmed


3.Subset of uterine anomalies-ashermans
by sonographic and histologic
syndrome,uterine myoma,mullerian duct
examination(ASRM 2008) anomalies
Completion curettage

4.Endocrine abnormalities-luteal phase defect


and PCOS

MANAGEMENT OF CERVICAL
CERVICAL INSUFFICIENCY INCOMPETENCE CERCLAGE PROCEDURES
Cervical incompetence Sonography for viability and exclude anomalies Mc Donalds-simpler,pursestring suturing of the
cervix
Painlesscervical dilatation in the second Test for Chlamydia and gonorrhea
trimester with prolapse of the BOW Shirodkar-transverse incision done at the mucosa
Cerclage-done at 12-14 weeks age of gestation
Diagnosed by ultrasound-funneling at the Transabdominal cerclage-suture is placed at the
internal os uterine isthmus
Contraindicationsto cerclage-bleeding,uterine
contractions,ruptured membranes
Riskfactors:Cervical trauma due to D and C,
conization,cauterization,amputation,DES Typesof cerclage:prophylactic(cervix closed) and
exposure rescue cerclage(cervix open)

OUTCOME OF ECTOPIC
COMPLICATIONS OF CERCLAGE ECTOPIC PREGNANCY PREGNANCIES
Membrane Rupture Ampulla is the most frequent site followed by the 1.TUBAL RUPTURE- earliest (isthmus) later
isthmus. Rarest site-previous CS scar (interstitial)
Preterm labor
Heterotopic
pregnancy-intrauterine and ectopic 2.TUBAL ABORTION-fimbrial and ampullary
pregnancies coexisting
Hemorrhage
3.PREGNANCY FAILURE WITH RESOLUTION
Highestrisk for tubal implantation-previous
Infection
tubal surgery

Followed
by sexually transmitted disease- most
common chlamydia
CLINICAL MANIFESTATION DIFFERENTIAL DIAGNOSIS DIAGNOSIS
CLASSIC TRIAD- amenorrhea,vaginal Abortion SERUM BETA HCG- discriminatory index more
bleeding,abdominal pain Infection than or equal to 1500mIU/ml expect to see a live
Myoma with degeneration intrauterine pregnancy.If no intrauterine
pregnancy is seen :failed uterine
PELVIC EXAMINATION-cervical motion Molar pregnancy pregnancy,complete abortion,ectopic pregnancy
tenderness,fullness of the culde sac,tender boggy Ovarian tumor in complication
mass in the adnexa,uterus slightly enlarged
Salpingitis
DOUBLING
TIME IN NORMAL
Appendicitis PREGNANCY-48 hours
Signs of diaphragmatic
Cystitis,renal stone,gatroenteritis
irritation,hypotension,tachycardia,vertigo,syncop
e
Passage of decidual cast-endometrium of
pregnancy(Arias stella reaction)

TRANSVAGINAL SONOGRAPHY OF
DIAGNOSIS ECTOPIC PREGNANCY LAPAROSCOPY
SERUM PROGESTERONE Trilaminar endometrium Diagnostic-direct visualization of the Fallopian
tube
>25 ng/ml-excludes ectopic pregnancy Complex adnexal masses
Therapeutic-surgical management
<5ng/ml-non living intrauterine pregnancy or Placental
blood flow within the periphery of the
ectopic pregnancy mass-RING OF FIRE pattern

Anechoic or hypoechoic fluid (hemoperitoneum)

TREATMENT OPTIONS SURGICAL MANAGEMENT INTERSTITIAL PREGNANCY


MEDICAL-(Methotrexate)folic acid antagonist SALPINGOSTOMY-unruptured, <2 Implantation
in the proximal segment within the
cms.,incision done with removal of the products muscular wall
and leave it open
CRITERIA:
Incorrectly called cornual pregnancy
SALPINGOTOMY- unruptured,incision done
1.low initial beta HCG (<1000 mIU/ml)
with removal of the products and suture the
2.small ectopic size(3.5 cms) Management:cornual resection or cornuostomy
incision
3.absent fetal heart activity

4.unruptured
SALPINGECTOMY-tubal resection
MANAGEMENT OF OVARIAN
OVARIAN PREGNANCY PREGNANCY
SPIEGELBERG CRITERIA: Ovarian wedge resection
1.ipsilateral tube is intact and distinct from the
ovary Cystectomy
2.ectopic pregnancy occupies the ovary
3.ectopic pregnancy is connected by uteroovarian Oophorectomy
ligament to the uterus
4.ovarian tissue can be identified histologically
Methotrexate injection
amid placental tissues
GESTATIONAL TROPHOBLASTIC
DISEASE
DEPARTMENT OF OBSTETRICS AND GYNECOLOGY
FEU-NRMF INSTITUTE OF MEDICINE
REFERENCE: WILLIAMS OBSTETRICS 24th EDITION

GTD CLASSIFICATION (FIGO,2002)


Spectrum of placental-related tumors: HYDATIDIFORM MOLE
Molar Complete
Nonmolar Partial Benign

GESTATIONAL TROPHOBLASTIC NEOPLASIA


(Malignant GTD/Persistent GTD)
Invasive Mole - Molar

Choriocarcinoma

Placental Site Trophoblastic Tumor- Non

Epithelioid Trophoblastic Tumor Molar HYDATIDIFORM MOLE

MOLAR PREGNANCY

MOLAR PREGNANCY:
HISTOLOGIC ABNORMALITY COMPLETE H-MOLE: GROSS COMPLETE H-MOLE: HISTO
Normal H-mole

Chorionic vIlli:
Vesicles of variable sizes with thin pedicles Generalized edema and cistern formation
Trophoblastic proliferation
Edema of villous stroma
COMPLETE H-MOLE: KARYOTYPE PARTIAL H-MOLE

Diploid, paternal origin

85%
are 46 XX (both from father)
ANDROGENESIS

Someare 46 XY (dispermic fertilization or


Dispermy)

PARTIAL H-MOLE: GROSS PARTIAL H-MOLE: HISTO PARTIAL H-MOLE: KARYOTYPE


Some elements of fetal tissues Slowly progressive Triploid: 69 XXX, 69XXY, 69XYY (diandry)
Less advanced or focal hydatidiform changes swelling of stroma of
avascular chorionic
Onematernal and two paternal haploid sets of
villi
chromosomes (dispermy)
Normal vascular villi
with functional fetal-
Fetusis nonviable with multiple malformations
placental circulation
or with severe growth restriction

TWIN MOLAR PREGNANCY DIAGNOSIS


Completediploid molar pregnancy + a normal
pregnancy (rare)

Differentiated to Partial Mole by fetal


karyotyping

Higherrisk of preeclampsia, hemorrhage and


subsequent gestational trophoblastic neoplasia**
THECA-LUTEIN CYSTS (OVARY) RISK FACTORS CLINICAL PRESENTATION
Age extremes of age (adolescent and > 40
Common in complete mole amenorrhea
years old)

Prior Molar Pregnancy Complete Mole- 1.5% Nausea and vomiting (maybe significant)
Smooth, yellow surface,
lined by lutein cells Partial Mole 2.7% Uterine bleeding (58%)
Overstimulation from Prior Molar Prenancy 23% Enlarged uterus (50% of cases in complete mole) -
hCG
OCP use soft
Previous miscarriage No fetal motion
Torsion, infarction,
hemorrhage Smoking Thyrotoxicosis (possible but not common) low
Regressed after evacuation Vitamin deficiencies TSH and high FT4
Only indication for
Increased paternal age

oophorectomy infarction Early onset preeclampsia hypoxic trophoblast


after untwisting Strongest risk factor releases anti-angiogenic factors endothelial
damage

TROPHOBLASTIC DEPORTATION OR
EMBOLIZATION PATHOLOGICAL DIAGNOSIS PATHOLOGIC DIAGNOSIS
Escape of trophoblast during molar evacuation Histological characteristics Complete Mole diploid (P57KIP2 negative)
going to the pulmonary parenchyma: Determination of Differing Ploidy, Diploid PartialMole - triploid (P57KIP2 positive)
(complete), Tri ploid (Partial) Spontaneous abortion with Hydropic
Acute pulmonary embolism or edema Immunostaining (P57KIP2) Nuclear protein degeneration - diploid (P57KIP2 positive)
Fatalities uncommon but possible Complete Negative
Partial Positive
Non Molar hydrophic pchange positve

MANAGEMENT:
MANGEMENT: PREOPERATIVE MANGEMENT: INTRAOPERATIVE SUCTION CURETTAGE
Large bore Intravenous Catheter(s) TREATMENT OF
Chest X-ray CHOICE
Regional or General Anesthesia Adequate anesthesia
MRI or CT scan: lung lesions or if with
Oxytocin 20 units in 1L of D5LR Blood availability
extrauterine disease (liver or brain) if cervix is closed (osmotic
Karman Cannula size 10, 12 or 14 dilators)
CBC
Sonography machine Oxytocin AFTER
ABO typing and Rh screen evacuation of most of the
Other uterotonics: molar tissues
Liver enzymes
Thorough gentle curettage
Methylergonovine - .2mg/ml every 2 hours with sharp curette AFTER
Baseline serum hCG
Carboprost (PGF2) 250 ug/ml every 15.90 minutes myometrium has
Creatinine contracted
Misoprostol (PGE1) 200 mg/tab (800-1000 mg once)
Intraoperative
Consider hygroscopic dilators ultrasonography*
MANAGEMENT: OTHER METHODS OF
COMPLICATIONS: INTRAOPERATIVELY TERMINATION POSTOPERATIVE:
Bleeding uterotonics, surgical methods Labor induction or hysterotomy- NOT done Anti D Immunoglobulin if Rh D NEGATIVE
( blood lossand persistent trophoblastic disease) Initiate Contraception
Trophoblastic deportation respiratory OCP or medroxyprogesterone acetate (poor
insufficiency, pulmonary edema, embolism Hysterectomy with ovarian preservation - if no compliance)
further pregnancy is desired IUD perforation
Barrier high failure rates
Do not remove theca lutein cyst Review Pathology Report
Aspiration of large cyst to minimize pain or Median time of Beta HCG resolution: Partial 7

torsion weeks and Complete 9 weeks


Serum Beta HCG Surveillance within 48 hrs,
weekly until undetectable the monthly for 6
months

RISK FACTORS FOR DEVELOPING GTN GESTATIONAL TROPHOBLASTIC


FOLLOWING MOLAR EVACUATION: ANTECEDENT PREGNANCIES TO GTN NEOPLASIA
H mole 50% Clinical Findings:
Complete 15-20% Partial 1-5% Aggressive myometrial invasion
Abortion/TubalPregnancies 25%

Preterm or Term Pregnancies 25% Strong tendency to metastasize


Older Age
Irregular bleeding with uterine subinvolution
Beta HCG > 100,000 u/ml Diagnosed only by persistent elevation of serum
hCG in most cases
Uterine size large for gestational age Myometrial Perforation intraperitoneal hemorrhage
Some with lower genital tract metastasis or distant
Theca Lutein cyst >6cm metastasis

Slow decline of HCG

Myometrial nodules of hypervascularity by postevacuation ultrasound

HISTOLOGIC CLASSIFICATION OF GTN INVASIVE MOLE INVASIVE MOLE


INVASIVE MOLE Formerly called Deep penetration into
CHORIOCARCINOMA Chorioadenoma the myometrium
destruens Almost always arise
PLACENTAL SITE TROPHOBLASTIC TUMOR
Excessive from a partial or
EPITHELIOID TROPHOBLASTIC TUMOR
trophoblastic complete mole
overgrowth with Locally invasive but
* The diagnosis of GTN is usually based on extensive tissue lacks the tendency for
persistent elevation of serum hCG level without invasion by widespread
confirmation by pathological study. Management trophoblastic cells and metastasis
is NOT directed by histologic findings. whole villi
GESTATIONAL CHORIOCARCINOMA GESTATIONAL CHORIOCARCINOMA GESTATIONAL CHORIOCARCINOMA
Extremely malignant carcinoma of chorionic Rapidly growing Involvement of
epithelium Invades myometrium cytotrophoblasts and
and blood vessels syncitial elements (one
2/3 follow a normal delivery, 1/3 follow molar hemorrhage and or the other may
gestation necrosis predominate)
Dark red or purplish,
Considered anytime there is persistent Columns or sheets of
ragged and friable
trophoblasts penetrating
bleeding after any pregnancy event Early bleeding,
myometrium and blood
sloughing and infection
with involvement of vessels
endometrium Plexiform or disorderly
Dark irregular nodules arrangement
with involvement of No villous patterns
peritoneum

GESTATIONAL CHORIOCARCINOMA:
METASTASIS
Early and Hematogenous (affinity of
trophoblasts to blood vessels)
Lungs (75%), Vagina (50%)

Vulva, kidneys, brain, bowel

Ovarian theca lutein- cysts seen in over 1/3 of


metastatic cases

GTN: CLINICAL COURSE FIGO 2002 STAGING WHO PROGNOSTIC SCORING, 2002
Irregular
bleeding with uterine Stage l- confined to uterus
subinvolution Stage ll-extends outside uterus but limited to
genital structures (adnexae, vagina, broad
Bleeding: continuous or intermittent ligament)
Stage lll-extends to lungs with or without known
Intraperitoneal
hemorrhage: myometrial genital involvement
perforation by invading trophoblasts
Stage lV-other metastatic sites

May
initially present as vulvar or vaginal
metastasis or other distant metastasis

Choriocarcinoma: fatal without treatment


WHO PROGNOSTIC SCORING, 2002 GTN: TREATMENT GTN: TREATMENT
Bestmanged by Oncologist Subsequent chemotherapy is based on serial
Singleagent chemotherapy: nonmetastatic or low hCG levels
6: low risk disease treatable by single risk disease Combination chemotherapy for high risk cases:

chemotherapeutic agent Methotrexate (less toxic) EMA-CO: Etoposide, Methotrexate, Actinomycin D,


Actinomycin D Cyclophosphamide, Oncovin(Vincristine)
7: high risk disease requiring combination
Risk factors for additional chemotherapy beyond Surgery, Radiotherapy
chemotherapy
first course:
Metastatic disease
Single-day methotrexate infusion
Complete mole histology

SUBSEQUENT PREGNANCY SAMPLE QUESTIONS:


Surveillance duration: (minimum) 1.A 21 y/o G1P0 PU 14 weeks consulted because 2.39y/o G4P0 consulted because of recurrent
6 months-molar pregnancy of minimal vaginal spotting for 2 weeks.No FHT pregnancy losses at 8-10 weeks AOG.She
1 year GTN heard on doppler.Cervix closed ,uterus 2 months mentioned that her previous pregnancies were all
2 years if GTN is with metastasis size,no adnexal mass nor tenderness.What is the anembryonic.Which of the following is the most
Pregnancy can be achieved after successful type of abortion? possible cause of her miscarriages?
treatment a.Threatened a.chromosomal anomalies

b.Complete b.endocrine
Sonographic evaluation on early pregnancy and
c.missed c.incompetent os
subsequently
Histopathology of placenta or products of d.incomplete d.infections
conception
Serum HCG determination 6 week postpartum

3.26y/o G3P0 PU 14 weeks had an ultrasound 4.30 y/o G1P0 12 weeks AOG consulted because 5.A 35 y/o G4P3 PU 22 weeks has an ultrasound
finding of live intrauterine pregnancy with of vaginal bleeding and foul smelling discharge result of IUFD 14 weeks.Cervix is closed and
funnelling of the internal os.Previous after an induced abortion.Temp-38 C,cervix open long.Uterus is enlarged to 3-4 months size.Which
pregnancies were terminated at 4months and 5 and tender on wriggling,uterus slightly enlarged of the following is NOT part of the work ups of
months respectively.The BEST management for andtender,adnexa bilaterally tender.What is the this patient?
the patient is BEST sequence of management for this patient? a.fasting blood sugar
a.bed rest and tocolytic a.paracetamol , completion curettage,antibiotics b.prothrombin time
b.tocolytic and progesterone b.broad spectrum antibiotics ,paracetamol, c.VDRL
c.tocolytic and cerclage completion curettage d.chest x ray
d.cerclage and progestin c.broad spectrum antibiotics ,paracetamol,explore
lap
d.completion currettage,antibiotics,paracetamol
6.21y/o G2P0 8 weeks AOG ,Preg test 7.21y/o G1P0 5 weeks amenorrhea has a positive 8.G1P0 6 weeks complains of vaginal spotting
positive,consulted because of vaginal spotting pregnancy test and an ultrasound result of and abdominal pain.cervix tender on
and abdominal pain.cervix closed tender on thickened endometrium to consider early wriggling,uterus slightly enlarged,with
wriggling,uterus slightly enlarged,with a pregnancy versus ectopic pregnancy.Which of the tenderness on the left adnexa.TVS revealed a 2
palpable vauge mass on the right tender on following findings will confirm ectopic cm mass on the left adnexa with ring of fire
palpation,left adnexa negative for mass and pregnancy? pattern and fetal heart beats.No free fluid seen in
tenderness.Which of the following is the best to a.progesterone assay -30ng/dl the culde sac.What is the best management for
arrive at a diagnosis? b.beta HCG of 1800 iu/ml
the patient?
a.beta HCG a.medical with methotrexate
c.beta HCG determination 0f 800 increasing to
b.transvaginal ultrasound 1800 in 48 hrs b.laparoscopic salpingostomy

c.progesterone assay d.arias stella reaction on dilatation and curettage c.laparoscopic salpingotomy

d.culdocentesis d.laparotomy,salpingectomy

9.Which of the following is not a part of the 10.Which of the following patients will have the 11.Which of the following is NOT is gestational
spigielberg criteria for the diagnosis of Ovarian highest risk for development of H mole? trophoblastic neoplasia?
pregnancy? a.22 y/o filipino a.complete mole
a.ipsilateral tube is intact and distinct from the b. thailander with previous partial mole b.placental site trophoblastic tumor
ovary
c.american smoker c.invasive mole
b.ectopic pregnancy occupies the ovary
d.G3P0 ,hypertensive d.choriocarcinoma
c.ectopic pregnancy is connected to the uterus by
the infundibulopelvic ligament
d.ovarian tissues can be identified histologically
amid placental tissues

12.Which of the following is found in partial 13.G1P0 PU 15 weeks has an ultrasound result 14.A 24y/o G3P2 has a complete mole at 14 weeks
mole? of snowstorm pattern on ultrasound.Beta HCG AOG with bilateral asymptomatic theca lutein
a.beta HCG of more than 100,000 -120,000IU/ML.The BEST management for the cyst measuring 8 cms.The management for the
b.uterus is larger than AOG
patient is ovarian cyst is
a.methotrexate a.expectant
c.mistaken as missed abortion
b.dilatation and curettage b.oophorocystectomy
d.P57Kip 2 negative
c.suction curettage c.aspiration

d.hysterotomy d.oophorectomy
15.Which of the following is TRUE of gestational 16.36y/o G7P5 underwent dilatation and 17.A G3P3 postpartum for 3 months complains of
choriocarcinoma? curettage for a septic abortion .She complained of vaginal bleeding since her last delivery.On
a.Majority follows an H mole severe abdominal pain 12 hours after.Abdominal examination ,a bluish mass which bleeds to touch
examination,with muscle guarding,with rebound is noted on the anterior vaginal wall and uterus
b.dilated villi microscopically
tenderness.What complication do you consider in is enlarged to 3 months size.What test will you
c.lesions in the vagina must be biopsied
this case? request to arrive at a diagnosis?
d.lungs is most common site of metastasis
a.uterine rupture a.beta HCG

b.uterine perforation b.transvaginal ultrasound

c.endometritis c.biopsy of the mass

d.embolism d.dilatation and currettage

18.40y/o G4P3 16 weeks AOG came back with an 19.What is the most common predisposing factor 20.What will differentiate a twin molar pregancy
ultrasound result showing intrauterine cystic for a tubal pregnancy? from a partial mole?
masses exhibiting snowstorm pattern.Beta a.previous infection a.beta HCG level
HCG-120,000 IU/ml.What is the best
b.tubal surgery b.p57kip 2
management for this patient ?
c.endometriosis c.fetal karyotyping
a.methotrexate
d.smoking d.ultrasound findings
b.dilatation and curettage

c.suction currettage

d.hysterectomy

RISK FACTORS
PLACENTA PREVIA CLASSIFICATION
Theplacenta goes before the fetus in the birth 1.PLACENTA PREVIA-internal os is partially or 1.Multifetal gestation
canal totally covered by the placenta 2.Maternal age
3.Multiparity
2.LOW LYING-implantation in the lower uterine
Trophotropism-with greater upper uterine flow 4.Prior Cesarean section-MOST COMMON
segment but the edge does not reach the internal
the placental growth is more towards the fundus os and remains outside a 2 cm wide perimeter 5.Cigarette smoking

6.Elevated Prenatal Screening MSAFP levels


Placental
migration is a misnomer MARGINAL PREVIA-placenta at the edge but
We can wait till 34 weeks for trophotropism to does not overlie it
occur
VASAPREVIA-fetal vessels course through the
membranes and present at the cervical os
CLINICAL FEATURES DIAGNOSIS MANAGEMENT
1.Painless bleeding-MOST characteristic event 1.Sonography-transabdominal,transvaginal PRETERM/NO PERSISTENT ACTIVE
(more accurate) and transperineal BLEEDING
2.abnormal adherence of the placenta -observation,tocolytic,steroids

2.MRI-expensive

Placenta accreta -ADHERENT 3.IE under double set up-done in the operating NEAR TERM/NOT BLEEDING
Placenta Increta -INVADES room and with preparation for emergency CS -scheduled cesarean section(36-37 weeks)
Placenta percreta-PENETRATES Low transverse(cutting through the placenta))

Vertical incision

For anterior placenta previa with a previous CS


scar-increase likelihood of placenta accreta

CLINICAL PRESENTATION AND


PLACENTA ACCRETA DIAGNOSIS MANAGEMENT
Partial or total absence of decidua basalis Hemorrhage together with previa or third stage Multidisciplinary approach
bleeding Prepare blood for transfusion
Imperfectdevelopment of the fibrinoid layer or Tertiary Hospital
nitabuchs layer SONOGRAPHY/COLOR MAPPING Elective CS after 36 weeks
-absent sonoluscent space between the placenta Classical CS with Hysterectomy
RISK FACTORS;placenta previa and previous and decidua
Do not attempt to remove the placenta
cesarean section,widespread use of MSAFP and uterine serosa bladder wall interface with
Other Option:Leave the placenta and give
HCGscreening for aneuploidies retroplacental vessels is <1mm,large methotrexate (not prepared for hysterectomy/
intraplacental lacunae desirous of another pregnancy)
MRI

ABRUPTIO PLACENTA RISK FACTORS CLINICAL MANIFESTATION


Premature separation of the normally implanted PRIOR ABRUPTIO most common Sudden onset abdominal pain
placenta Increasing age and Parity Vaginal bleeding
Hypertension
Uterine tenderness
Chorioamnionitis
Total or partial Hypertonic uterine contractions
PPROM
Multifetal pregnancy Non reassuring fetal heart rate
External hemorrhage or concealed (more DIC) Low birthweight Fetal death
Hydramnios
Cigarette smoking
Thrombophilias
Cocaine
Uterine leiomyoma
DIAGNOSIS MANAGEMENT COMPLICATIONS
Clinical Manifestation CESAREAN DELIVERY-compromised fetus,dead Hypovolemic shock
fetus but with severe maternal hemorrhage Consumptive coagulopathy-fibrinogen level of
Sonography-negative result does not rule out Transverse lie,or with previous hysterotomy scar less than 150 mg/dl
abruptio VAGINAL DELIVERY- dead fetus but with Couvelaire uterus-( uteroplacental
minimal hemorrhage apoplexy).Seldom causes atony and not an
Early amniotomy-to hasten delivery and decrease indication for hysterectomy
MRI-highly sensitive
thromboplastin infusion in the circulation Acute kidney injury

SHEEHANS syndrome-failure of
Elevated D dimers
lactation,amenorrhea,breast atrophy,loss of pubic
and axillary hair,hypothyroidism,adrenal cortical
insufficiency

POSTPARTUM HEMORRHAGE UTERINE ATONY EVALUATION AND MANAGEMENT


Traditionally term as blood loss of 500 ml or more RISK FACTORS: Inspect for lacerations
Commonly caused by atony and lacerations Primiparity/multiparity Inspect the placenta
Atony-uterus is soft and boggy Overdistended uterus Uterine massage

Lacerations-persistent bleeding with a uterus Labor abnormalities


that is well contracted Uterotonics-oxytocin,methylergonovine,Prostaglandin
Labor induction/augmentation
E2 (Dinoprostone)
Types:Early postpartum hemorrhage-within 24 Prior postpartum hemorrhage PGF2(carboprost),misoprostol(PGE1)
hours
Late postpartum hemorrhage-bleeding after 24
Uterine packing and balloon tamponade
hours

Surgical procedures-compression sutures,pelvic vessel


ligation,angiographic embolization,hysterectomy

OTHER CAUSES OF POSTPARTUM


UTERINE INVERSION MANAGEMENT HEMORRHAGE
Risk factors: 1.Obsterician and anesthesiologist(Halothane/ 1.LACERATIONS

1.fundal placenta enflurane) Persistent


vaginal bleeding with well contracted
2.uterine atony 2.Blood uterus
3.cord traction before placental separation 3.large bore IV lines MANAGEMENT:SUTURING

4.adherent placenta 4.if placenta is still attached,removed the


placenta and reposition the uterus under 2.HEMATOMAS-vulvar pain,tense and fluctuant
anesthesia or tocolytic such as mass with discolored skin
terbutaline,Mgso4,nitroglycerine MANAGEMENT:evacuation and suturing
5.Once repositioned,start oxytocin

6.If reposition fails surgical management

Huntingtons and Haultain


UTERINE RUPTURE MANAGEMENT OF HEMORRHAGE QUESTIONS:
1.Primary-occurs in unscarred or intact uterus 1.Fluid resuscitation 1. G2P1 PU 32 weeks consulted because of
2.Blood replacement vaginal bleeding.No uterine contractions
2.Secondary-with preexisting uterine a.Packed RBC-1 unit increases hematocrit by 3-4
noted.FHT-140/minute.The initial procedure that
incision,injury or anomaly vol % you will perform is
a.gentle speculum examination
b.Platelets-indicated if platelet count is below
50,000 b.internal examination
CAUSES;
c.Fresh frozen plasma-use for consumptive and c.transvaginal ultrasound
1.Pre
existing uterine injury example CS
dilutional coagulopathy d.transabdominal ultrasound
2..Uterineinjury or abnormality incurred during
current pregnancy d.Cryoprecipitate and fibrinogen-DIC,fibrinogen

(example:oxytocin,version,overdistended uterus) deficiency

2.36 y/o,G3P2 PU 35 weeks complained of vaginal 3.36 y/o G4p3 PU 33 weeks, Previous CS 2x has 4.G1P1 postpartum 2 hrs was brought by the
bleeding and abdominal pain.BP-150/100 anterior placenta previa.What condition will you midwife because of profuse bleeding.She
mmhg.Uterine contractions were noted every 1-2 have to rule out in this patient prior to a repeat delivered at home to an 8 lbs. baby.Placenta was
minutes 60 secs in duration.Her last ultrasound cesarean section? spontaneously expelled after 10 minutes.On
2 weeks ago was normal.IE-cervix closed a.vasa previa examination ,uterus is contracted and palpated
uneffaced.What is the diagnosis? below the navel.What is the diagnosis?
b.uterine dehiscence
a.preterm labor a.uterine atony
c.placenta accreta
b.vasa previa b.retained placenta
d.adhesions
c.placenta previa c.lacerations

d.abruptio placenta d.uterine inversion

5.32y/o G3P2 PU 30 weeks complains of 6.36y/o G3P2 PU 36 weeks complained of vaginal 7. What is the MOST dreaded complication of
moderate vaginal bleeding.Ultrasound done bleeding and abdominal pain.BP-120/80 mmhg abruptio placenta?
revealed a placenta totally covering the os.What but she is a known hypertensive for 2 a.Hypovolemia
is the management for this patient? years.Uterus is woody with no FHT heard by b.Septicimia
a.Bed rest and give tocolytic doppler.cervix is 3 cms dilated 1cm long , BOW
intact,cephalic station -1,with minimal c.Embolism
b.Bed rest,tocolytic,progesterone
bleeding.What is the BEST management ? d.DIC
c.Bed rest ,tocolytic,steroids
a.expectant
d. Steroids,tocolytic and deliver after 48 hrs.
b.amniotomy

c.induce with oxytocin

d.immediate CS
8.A 34y/o G3P2 postpartum 1 hr was brought by 9.A G1P1 complains of vaginal bleeding 2 hours 10.36 y/o G3P2 PU 37 weeks previous CS 2x has
a midwife because of vaginal bleeding and after she delivered a 3.8kg baby via NSD .Uterus an ultrasound findings of anterior placenta
abdominal pain.On examination,a fleshy mass is soft and boggy palpated above the navel.What previa with absence of sonoluscent space between
was seen protruding out of the introitus,the is the initial management for this patient? the placenta and decidua.How should this patient
fundus of the uterus cannot be palpated a.bimanual uterine compression be managed?
abdominally.What is cause of this condition? b.ice pack a.CS with manual removal of the placenta
a.age and parity b.CS ,leave the placenta in situ,methotrexate
c.uterine artery ligation
b.strong traction of the cord c.CS,removal of placenta ,hysterectomy
d.hysterectomy
c.size of the baby d.CS with hysterectomy with placenta in situ
d.length of labor

11.A G3P3 postpartum 6 months ago was 12. 36 y/0 G4P4 patient had a CS due to abruptio 13.G3P2 PU 38 weeks Previous CS 2x was noted
selivered by NSD and complicated by atony .She placenta.The uterus was noted to be bluish with to have placenta invading the myometrium and
was transfused with 4 u PRBC.She has hematoma on the anterior and posterior wall and bladder serosa.What layer is defective in this
amenorrhea,failure to lactate and loss of pubic well contracted.What is the management? case?
hairs.What is the diagnosis? a.expectant a.decidua vera
a.ashermans b.uterine artery ligation b.nitabuchs
b.sheehans c.compression suture c.myometrium
c.simmonds d.hysterectomy d.peritoneum
d.PCOS

PRETERM LABOR
14. G1P0 PU 32 weeks has placenta partially 15.A patient who delivered by CS due to abruptio Preterm Birth-delivery before 37 completed
covering the os.What is the BEST management placenta was noted to have bleeding per vagina weeks
for this patient? and at the incision sites.Platelet
a.wait for spontaneous labor count-90,000,Prolonged prothrombin time and CAUSES OR PRETERM BIRTHS:
partial thromboplastin time.What is the BEST
b.Give steroids and do CS after 48hrs 1.Spontaneous
component therapy for her?
c.repeat the ultrasound at 35 weeks 2.PPROM
a.whole blood
d.schedule for CS at 38 weeks 3.Maternal and fetal indications
b.PRBC
4.Twins and higher order births
c.fresh frozen plasma

d.platelet concentrate
ANTECEDENTS AND
SPONTANEOUS PRETERM PPROM CONTRIBUTING FACTORS
UTERINE DISTENSION Spontaneous rupture of fetal membranes before Threatened abortion
MATERNAL AND FETAL STRESS 37 weeks and before labor onset Cigarette smoking,inadequate maternal weight
INFECTION-Gardnerella Increase levels of membrane proteases gain,illicit drug
vaginalis,fusobacterium,mycoplasma,ureaplasma Infection causes membrane weakening leading to Genetic factor
urealyticum rupture Birth defects

Periodontal disease

Interval between pregnancies

Prior preterm birth-MOST COMMON

Infection-bacterial vaginosis

DIAGNOSIS MANAGEMENT OF PPROM TOCOLYTIC AGENTS


Symptoms-regular contractions 34 WEEKS OR MORE-induction of labor BETA ADRENERGIC RECEPTOR AGONIST
1.Ritodrine
Cervical
change in dilatation and length-less 32-33
WEEKS-expectant,antibiotics , grp B strep. 2.Terbutaline
than 25 mm prophylaxis,steroids MAGNESIUM SULFATE-use for neuroprotection
PROSTAGLANDIN INHIBITORS(Indomethacin)
Ambulatory Uterine Testing 24-31
WEEKS-expectant,steroids,Grp B strep. CALCIUM CHANNNEL BLOCKERS-Nifedipine
Prophylaxis,antibiotics,tocolytics(?)
ATOSIBAN
Fetal Fibronectin->50 ng/ml NITRIC OXIDE DONORS
Less
than 24 WEEKS-patient
counselling,expectant,Grp.B strep
prophylaxis,antimicrobial

CORTICOSTEROID THERAPY POSTTERM PREGNANCY COMPLICATIONS


Given at 24 to 32 weeks Also
called prolonged pregnancy extended 1.Oligohydramnios

Can also be given at 32-34 weeks pregnancy


2.Macrosomia

TYPES; PATHOPHYSIOLOGY:

Betamethasone-12 mg every 24 hours for 2 doses 1.Postmaturity syndrome 3.Medical and obstetrical complications
Dexamethasone-6 mg every 12 hours for 4 doses 2.placental dysfunction
3.fetal distress and oligohydramnios

4.fetal growth restriction


MANAGEMENT GROWTH RESTRICTION RISK FACTORS AND ETIOLOGIES
1.UNFAVORABLE CERVIX 1.Symmetrical-proportionately small,early Small mothers
a.cervical ripening agents-PGE2 and membrane insult,decrease in cell number abdominal size. Poor maternal weight gain
stripping or sweeping Causes:chemical exposure,viral infection,cellular Poor nutrition
maldevelopment Social deprivation
42 weeks with fetal compromise and oligohydramnios
Drugs and teratogen
-INDUCE
2.Asymmetrical-disproportionately lagging Infection
abdominal growth,late insult such as placental
42 weeks ,no complications ,favorable cervix-INDUCE Fetal malformations
insufficiency(hypertension)
Genetic abnormalities and multifetal pregnancy

42 weeks ,no complications,unfavorable cervix- Placenta and cord abnormalites


FETAL SURVEILLANCE WITH AF ASSESSMENT

DIAGNOSIS MANAGEMENT MACROSOMIA


Uterine Fundal height-18-30 weeks the fundic >34WEEKS-doppler/NST/BPS weekly and fetal Newborn who weighed >4500 grams
height is almost equal as the age of gestation growth every 3-4 weeks RISK FACTORS:
1.Obesity
Ultrasound DELIVER IF WITH absent or reversed end 2.Diabetes-gestational ,type2-MOST COMMON
diastolic flow,oligohydramnios,non reassuring
3.postterm gestation
fetal status,or with obstetrical indication to
Amniotic fluid measurement 4.multiparity
terminate,no growth in the biometry
5.large size parents
Doppler velocimetry 6.advancing maternal age
>24but less than 34 WEEKS-SAME as above
except you will give steroids 7.previous macrosomic infant

8.racial and ethnic factors

MANAGEMENT QUESTIONS:
DIABETIC with EFW-4250 grams cesarean 1.Which of the following will put the patient at 2.G3P2(0-2-0-0) PU 32 weeks has watery vaginal
section the highest risk for the development of Preterm discharge.Nitrazine paper test positive(yellow to
Non diabetic.->5000 grams Labor? blue).What is the management?
Prophylactic Induction of labor-proposed to a.multiparity a.tocolytic

reduce cesarean delivery and shoulder dystocia b.smoking b.steroid andtocolytic


HOWEVER,studies showed it increases risk of c.prior preterm birth c.expectant,steroid and ampicillin
CS without improved neonatal outcomes
d.infection d.steroid,tocolytic.ampicillin
3.Which of the following findings is indicative of 4.30 y/o G4P3 PU 30 weeks was seen.Pregnancy 5.A 35 y/o G3P2 PU 41 weeks has an ultrasound
preterm labor? test was positive at 4 weeks AOG.Fundic height findings of BPS-6/8 with AFI 4 cms.Cervix is
a.uterine contractions with closed cervix -24 cms FHT-140/min.What is the assessment of closed and 1.5 cms long cephalic station 0.What
this pregnancy ? is the best management ?
b.cervical length of 20mm
a.normal pregnancy a.hydrate patient
c.fibronectin -20 ng/ml
b.inaaccurate aging b.CST and induce if negative
d.hypogastric pain
c.intrauterine growth restriction c.close fetal surveillance

d. large for date pregnancy d.cesarean delivery

6.What is the most common risk factor for the 7.A G4P3 PU 32 weeks has a fundic height of 24 8.A G1P0 PU 38 weeks has a fundic height of 39
development of fetal macrosomia? cms.Biometry revealed a BPD /femur length cms.Estimated fetal weight by ultrasound is 4250
a.obesity compatible with 30 weeks and an abdominal grams.Her 75 gms OGTT revealed FBS -105 mg/
circumference compatible with 24 weeks dl and 2nd hr -160mg/dl.What is the
b.diabetes
AOG .Which of the following is the cause of this management?
c.multiparity
condition? a.wait for spontaneous labor
d.nutrition
a.genetic b.induce labor with prostaglandin
b.chemical exposure c.Wait for 39 weeks and induce with oxytocin
c.hypertension d.elective CS at 39 weeks
d.viral infection

DYSTOCIA
9.G2P1 PU 34 weeks,cephalic has a fundic height 10.G1P0 PU 42 weeks has an AFI-2cms.cervix COMMON CLINICAL FINDINGS WITH
of 26 cms.Doppler velocimetry is requested every closed ,uneffaced but soft.Which of the following INEFFECTIVE LABOR
week to monitor the fetus.Which of the following is the best to induce labor in this patient?? 1.Inadequate cervical dilatation and descent
findings will indicate severe fetal compromise? a.membrane sweeping Protracted-slow progress
a.increase resistance index b.oxytocin Arrested-no progress
b.diastolic notching c.prostaglandin Inadequate expulsive efforts
c.absent end diastolic flow 2.Fetopelvic disproportion
d.primrose oil
d.reversed end diastolic flow
Fetal size,pelvis,malpresentation or position
3.Ruptured membranes without labor
UTERINE DYSFUNCTION PROLONGATION DISORDER ACTIVE PHASE DISORDERS
HYPOTONIC- no basal hypertonus,uterine Prolonged Latent phase PROTRACTION DISORDERS:Treatment-Expectant
contractions have normal gradient >20 hours-Nullipara Protracted active phase-<1.2/hr in nullipara,,1.5 cm/
pattern(synchronous) but insufficient to dilate >14 hrs Multipara
hr in multipara
the cervix Protracted descent- <1 cm/hr in Nullipara,< 2cm/hr in
Treatment:Bed rest
Treatment: Oxytocin multipara
Exceptional Treatment:Oxytocin or cesarean
HYPERTONIC incoordinate,elevated basal ARREST DISORDERS:Treatment:CS if with CPD
delivery for urgent problems
tone,distorted pressure gradient Prolonged deceleration- >3 hrs N/ >1 M
Treatment: sedation Arrest in dilatation->2 hrs in Nulli and multi

Arrest in descent ->1 hr in Nulli and Multi

Failure in descent No descent in deceleration phase

REVISED GUIDELINES ON ARREST CAUSES OF UTERINE


SECOND STAGE DISORDERS DISORDERS DYSFUNCTION
Arrest of cervical dilatation- 6cms with Epidural anesthesia
PROLONGED SECOND STAGE OF LABOR membrane rupture for 4 hours with 200 Chorioamnionitis
montevideo units uterine contractions
Maternal position during labor-ambulation
Or 6 hrs if with no adequate contractions
Nulliparas- 2 hours /if with epidural-3 hours Birthing position-
upright,sitting,kneeling,squatting,30 degree
Prolongedsecond stage- 3 hrs for nulliparas elevation
Multiparas-1 hour/if with epidural- 2hours
without epidural and 4 hrs if with epidural Water immersion

PRECIPITOUS LABOR AND


DELIVERY PELVIC CONTRACTION CONTRACTED MIDPELVIS
Delivery of the fetus in less than 3 hours 1.INLET Causes transverse arrest of the head
Obstetric cojugate less than 10 cms,diagonal Interspinous diameter is less than 8 cms
Cervicaldilatation 5cms/hr in nullipara and 10 conjugate less than 11.5 cms and GTI less than Sum of the interpinous(10.5) plus posterior
cms /hr in multipara 12 cms sagittal diameter (5) is less than 13.5 cms
Associated with asynclitism ,early rupture of CLINICALLY:prominent spines,convergent side
membranes and descent doesnt occur until after walls,narrow sacrosciatic notch
Maternal and fetal effects:amniotic fluid
labor onset,less efficient uterine
embolism,uterine atony,abruptio,meconium,low
apgar score contractions,malpresentations,cord prolapse

TREATMENT;Discontinue
oxytocin,analgesic,tocolytic,general anesthesia
CONTRACTED OUTLET MUELLER HILLIS MANEUVER FACE PRESENTATION
Interischial tuberous diameter is 8 cms or less Fetalbrow and suboccipital region are grasped to
Often associated with midpelvic contraction the abdominal wall with the fingers and firm head is hyperextended , occiput is in contact with
pressure is directed downward along the axis of the fetal back and the chin (mentum) is
the inlet presenting
fetal face may present with the chin (mentum)
anteriorly or posteriorly, relative to the maternal
symphysis pubis

FACE PRESENTATION FACE PRESENTATION FACE PRESENTATION


Etiology Diagnosis
The occiput is the Prematurity
longer end of the
Marked enlargement of the neck or coils of cord Vaginal examination
head lever. The
about the neck may cause extension palpation of the distinctive facial features of the
chin is directly mouth and nose, the malar bones, and particularly
Anencephalic fetuses
posterior. Vaginal the orbital ridges (differentiate it from breech)
delivery is Contracted pelvis
Radiographic examination
impossible unless Very large fetus
the chin rotates Multiparous women
anteriorly
Hydramnios

FACE PRESENTATION FACE PRESENTATION BROW PRESENTATION


Mechanism of Labor Rarest presentation because it often converts to
Management face or occiput presentation
Descent - same factors as in cephalic presentations fetal head between the orbital ridge and anterior
internal rotation - the objective is to bring the chin under the fontanel presents at the pelvic inlet
Vaginal Delivery (Mentum Anterior)
symphysis pubis
Cesarean delivery (Mentum Posterior) fetal head occupies a position midway between
- same factors as in vertex presentations
flexion full flexion (occiput) and extension (mentum or
extension
face)
external rotation - results from the relation of the fetal body to
the deflected head
BROW PRESENTATION BROW PRESENTATION TRANSVERSE LIE
Diagnosis Mechanism of Labor
very small fetus and a large pelvis - labor is the long axis of the fetus is approximately
Abdominalpalpation - when both the occiput and generally easy perpendicular to that of the mother
chin can be palpated easily larger fetus - usually difficult Long axis forms an acute angle Oblique Lie

Persistent Brow vaginal delivery is difficult and (unstable)


Vaginalexamination palpation of the frontal management is same as face referred to as shoulder or acromnion presentation

sutures, large anterior fontanel, orbital ridges, the shoulder is usually on the pelvic inlet, with
eyes, and root of the nose the head lying on one iliac fossa and the breech
in another

TRANSVERSE LIE
TRANSVERSE LIE TRANSVERSE LIE
Etiology Diagnosis
Abdominal wall relaxation from high parity. Abdominal examination

Preterm fetus.

Placenta previa.
no fetal pole is detected in the fundus, ballottable
head is found in one iliac fossa and the breech in the
Abnormal uterine anatomy. other
Excessive amnionic fluid. (anterior) - a hard resistance plane extends across
Contracted pelvis. the front of the abdomen
(posterior)- irregular nodulations representing the
small parts are felt through the abdominal wall.

Palpation in transverse lie, right acromidorsoanterior position. A. First


maneuver. B. Second maneuver. C. Third maneuver. D. Fourth maneuver.

TRANSVERSE LIE TRANSVERSE LIE


Diagnosis Management
Indication for cesarean delivery

Vaginal examination vertical incision is recommended especially for


early stages of labor: the side of the thorax or the dorsoanterior presentation(Back down)
"gridiron" feel of the ribs
Advanced labor: the scapula and clavicle are palpated

CONDUPLICAT
O
CORPORE
Neglected shoulder presentation. A thick muscular band forming a pathological retraction ring
has developed just above the thin lower uterine segment. The force generated during a uterine
contraction is directed centripetally at and above the level of the pathological retraction ring.
This serves to stretch further and possibly to rupture the thin lower segment below the retraction
ring. (P.R.R. = pathological retraction ring.)
COMPOUND PRESENTATION COMPOUND PRESENTATION COMPOUND PRESENTATION

Causes
an extremity prolapses alongside the presenting Prognosis and Management
part or with both presenting in the pelvis preterm delivery, prolapsed cord, and traumatic
conditions that prevent complete occlusion of the
simultaneously pelvic inlet by the fetal head, including preterm obstetrical procedures
birth
Prolapsed arm alongside the head ascertain
whether the arm retracts out of the way with descent
of the presenting part, if it fails to retract and if it
appears to prevent descent of the head, the prolapsed
arm should be pushed gently upward and the head
simultaneously downward by fundal pressure

RISK FACTORS FOR PERSISTENT OCCIPUT


PERSISTENT OCCIPUT POSTERIOR POSITION POSTERIOR (POP) MORBIDITY ASSOCIATED WITH POP
Prolonged second stage of labor
Increased CS delivery and operative vaginal
delivery
Epidural analgesic
Transverse narrowing of the midpelvis Increased blood loss (vaginal delivery)
Nulliparity
Higher order vaginal lacerations (3rd and 4th
Greater fetal weight
degree lacerations)
Usually undergo spontaneous anterior rotation Prior Occiput posterior position delivery
followed by uncomplicated delivery

DELIVERY OF PERSISTENT OCCIPUT PERSISTENT OCCIPUT TRANSVERSE


POSTERIOR POSITION SHOULDER DYSTOCIA

1. Spontaneous Vaginal Delivery occiput tends to rotate to anterior position in the Head to body delivery time
Roomy pelvic outlet or relaxed perineum
absence of a pelvic architecture abnormality or Normal birth 24 seconds
2. Manual rotation to occiput anterior and spontaneous asynclitism.May use (Kielland forceps) Shoulder dystocia - > 60 seconds
delivery
Spontaneous delivery or delivery with outlet Fetal shoulder become wedged behind symphysis
Resistant vaginal outlet or form perineum
forceps.(simpsons) pubis and fail to deliver
3. Forceps or Vacuum delivery
Ineffective expulsive efforts EMERGENCY because the umbilical cord is
Must meet criteria for forceps or vacuum delivery compressed within the birth canal.
4. Cesarean section
Elongation of fetal head (molding/caput)
Head not engaged
SHOULDER DYSTOCIA PREDICTORS FOR SHOULDER DYSTOCIA SHOULDER DYSTOCIA
Management
Maternal Consequences
1. Increasing fetal weight risk factors:
Obesity Reduction in the interval of time from delivery of the
Postpartumhemorrhage - usually from uterine atony, Multiparity
vaginal and cervical lacerations

head to delivery of the body
Diabetes Mellitus and Gestational Diabetes Mellitus
initial gentle traction, assisted by maternal expulsive
Post term pregnancy
Fetal Consequences 75 % shoulder dystocia cases Birthweight >
efforts
Fetal morbidity and mortality (Neuromusculoskeletal 4000 grams Large episiotomy
injuries) 2. Intrapartum Factors: Adequate analgesic
Brachial Plexus Injury Prolonged second stage
Clavicular fracture/Humeral fracture/Rib Fracture Operative vaginal delivery
Hypoxia
Prior shoulder dystocia

SHOULDER DYSTOCIA
TECHNIQUES TO FREE THE ANTERIOR SHOULDER FROM ITS
IMPACTED POSITION BENEATH THE SYMPHYSIS PUBIS:

SHOULDER DYSTOCIA McRoberts maneuver


TECHNIQUES TO FREE THE ANTERIOR SHOULDER FROM ITS consists of removing the legs from the stirrups and sharply
Moderate POSITION
IMPACTED suprapubic pressure
BENEATH THE SYMPHYSIS PUBIS: flexing them up onto the abdomen
downward traction is applied to the fetal head. rotation of the symphysis pubis toward the maternal head,
Pressure is applied with the heel of the hand on the and a decrease in the angle of pelvic inclination
anterior shoulder pelvic rotation cephalad tends to free the impacted anterior
shoulder

Shoulder dystocia with


SHOULDER DYSTOCIA
SHOULDER DYSTOCIA impacted anterior TECHNIQUES TO FREE THE ANTERIOR SHOULDER FROM ITS
shoulder of the fetus. IMPACTED POSITION BENEATH THE SYMPHYSIS PUBIS:
The McRoberts A.The operator's hand is
maneuver. The introduced into the Woods corkscrew maneuver
maneuver consists of vagina along the fetal
The hand is placed behind the posterior shoulder of the fetus
removing the legs from posterior humerus,
and progressively rotating the posterior shoulder 180 degrees
the stirrups and sharply which is splinted as the
in a corkscrew fashion so the impacted anterior shoulder could
flexing the thighs up onto arm is swept across the
be released
the abdomen, as shown chest, keeping the arm
by the horizontal arrow. flexed at the elbow.
The assistant is also B. The fetal hand is
providing suprapubic grasped and the arm
pressure simultaneously extended along the side
(vertical arrow). of the face.
C. The posterior arm is
delivered from the
vagina
SHOULDER DYSTOCIA SHOULDER DYSTOCIA SHOULDER DYSTOCIA DRILL
1. Call for helpmobilize assistants, an
Deliberate fracture of the clavicle anesthesiologist, and a pediatrician. Initially, a
Rubins maneuver pressing the anterior clavicle against the ramus of the pubis to gentle attempt at traction is made. Drain the
free the shoulder impaction
Applying pressure on the posterior aspect of the anterior bladder if it is distended.
Hibbard Maneuver
shoulder towards the chest 2. A generous episiotomy (mediolateral or
pressure is applied to the fetal jaw and neck in the direction of the
maternal rectum, with strong fundal pressure applied by an episioproctotomy) may afford room posteriorly.
assistant as the anterior shoulder is freed 3. Suprapubic pressure is used initially by most
Zavanelli maneuver practitioners because it has the advantage of
cephalic replacement into the pelvis and then cesarean delivery. simplicity. Only one assistant is needed to
Cleidotomy provide suprapubic pressure while normal
cutting the clavicle with scissors or other sharp instruments downward traction is applied to the fetal head.
usually used for a dead fetus 4. The McRoberts maneuver requires two
Symphysiotomy assistants.

IF THE ABOVE MANEUVERS FAIL: COMPLICATIONS WITH DYSTOCIA TYPES OF BREECH PRESENTATION
5. Delivery of posterior arm FRANK- thighs are flexed and legs are extended
6. Woodscrew MATERNAL PERINATAL

7. Rubins manever Uterine rupture Fetalsepsis COMPLETE- thighs and legs are both flexed
IF IT FAILS Pathological retraction Caput succedaneum

8. Cleidotomy ring Molding INCOMPLETE(FOOTLING) one or both thighs


Fistula formation Nerve injury/fractures
or legs is /are extended and maybe prolapsed into
9. Zavanelli
the vagina
Pelvic floor injury Cephalhematoma
10. Symphysiotomy
Infection

Postpartum
hemorrhage

EXTERNAL CEPHALIC VERSION CONTRAINDICATIONS TO ECV


The fetus is turned in utero by manipulation of Any contraindications to labor
the maternal abdomen from a non cephalic to placenta previa,
cephalic non reassuring fetal heart rate,
Should be performed at more than 36 weeks IUGR
PREREQUISITES TO ECV Congenital anomalies (hydrocephalus)
Singleton,AOG- more than or equal to 37 weeks,no HIV patients
contraindications to labor,fetal well being established
prior to procedure, amniotic fluid and position of
fetus are known,facilities for immediate delivery
TECHNIQUE OF EXTERNAL
CEPHALIC VERSION TECHNIQUE
Must be performed in a facility equipped to FORWARD ROLL-each hand grasps one of the
perform emergency CS fetal poles and the buttocks are elevated from the
Sonographic evaluation to determine maternal pelvis and displaced laterally.The
presentation,amniotic fluid,exclude fetal buttocks are gently guided toward the fundus
anomalies,location of placenta while the head is directed toward the pelvis
External monitoring of fetal heart rate reactivity BACKWARD FLIP

Give anti D immunoglobulin for Rh negative


patients

PLANNED VAGINAL BREECH TYPES OF VAGINAL BREECH TECHNIQUE FOR VAGINAL BREECH
DELIVERY DELIVERY DELIVERY
Frank or complete breech at 36 weeks or more SPONTANEOUS BREECH DELIVERY- the Ensure adequate analgesia
with estimated fetal weight of 2500 grams to fetus is expelled entirely without traction or Spontaneous expulsion up to the navel with
4000 gms manipulation other than support of the newborn maternal pushing DONT PULL THE BREECH
Frank or complete breech at 31-35 weeks when PARTIAL BREECH DELIVERY-fetus is Facilitate rotation to sacrum anterior
estimated fetal weight is 1500-2500 grams delivered spontaneously up to the umbilicus and
Episiotomy when buttocks and anus are
BUT THE WOMANS WISHES IN COLLABORATION the remainder of the body is extracted with crowning
WITH THE ATTENDING PHYSICIANS operators traction and maneuvers
JUDGEMENT SHOULD DETERMINE WHICH If legs are not delivered spontaneously do
TOTAL BREECH DELIVERY- the entire body of
DELIVERY METHOD IS MOST APPROPRIATE PINARDS MANEUVER
the fetus is extracted by the obstetrician

TECHNIQUE OF VAGINAL BREECH


PINARDS MANEUVER DELIVERY

Inserttwo fingers along one extremity to the Support


the baby around the hips and ask the
knee woman to push until scapulae are visible
Abduction of the thigh away from the midline

Apply pressure on the popliteal fossa to flex the DONT


HOLD THE FLANK OR ABDOMEN,DO
legs and delivery of the foot NOT PULL THE BREECH

Maintain
flexion of the fetal head by keeping the
body below the horizontal
DELIVERY OF THE ARMS DELIVERY OF THE FETAL HEAD
Rotate the body to facilitate delivery of the arms MAURICEAU-SMELLIE VEIT MANEUVER
LOVESET MANEUVER
When the anterior shoulder and arm appear at Maintain the head in flexion
the vulva two fingers are applied on the
antecubital fossa to flex the arm and sweep it Place the attendants two fingers on the chin and
across the chest as if a cat is washing his face malar eminences

The assistant help by providing suprapubic pressure


Thebody of the fetus is then rotated in the as traction is applied by the primary health care
reverse direction to deliver the other shoulder provider
and arm

HEAD ENTRAPMENT
DELIVERY OF THE FETAL HEAD
USE OF PIPERS FORCEPS
The fetal body is elevated using warm towel Duhrssens Incision-2 , 10 o clock position
ADD 6 OCLOCK POSITION IF NOT SUCCESSFUL
Left blade is applied to the aftercoming head
Zavanelli maneuver-cephalic replacement
The right blade is applied with the body still
elevated Symphysiotomy

Embryogenesis of the Reproductive Tract

UTERUS
VULVAR ABNORMALITIES
Imperforate hymen
ABNORMALITIES OF
formed by the union of the 2 mullerian ducts fusion between the
sinovaginal bulbs with the
THE REPRODUCTIVE urogenital sinus.
Upper third of vagina-mullerian duct primary amenorrhea and
TRACT

Lower vagina urogenital sinus Cyclic pelvic pain

Bulging mass at the


introitus
Hematocolpos/hematometra

Treatment:cruciate incision
DEPARTMENT OF OBSTETRICS AND GYNECOLOGY of the hymen
FEU NRMF INSTITUTE OF MEDICINE
Vaginal Abnormalities VAGINAL ABNORMALITIES
UTERINE MALFORMATIONS
Vaginal agenesis Vaginal atresia
Failure of dissolution of the cell cord between Partial or complete Discovered by:
the urogenital sinus and mullerian tuburcle Associated with Rokitansky-Kuster-Hauser routine pelvic examination.
syndrome or androgen insensitivity cesarean delivery or during manual exploration
of the uterine cavity after delivery.

Longitudinal septate vagina


Obstetrical Significance
Results in miscarriage, ectopic pregnancy,
Transverse septate vagina rudimentary horn pregnancy, preterm delivery,
Results from faulty canalization of the fused fetal growth restriction, abnormal fetal lie,
mullerian anlage uterine dysfunction, or uterine rupture.

CLASS II UNICORNUATE UTERUS (CLASS II) CLASS III


increased incidence of infertility, endometriosis,
and dysmenorrhea.
Pregnancy in the normal-sized hemiuterus :
- preterm delivery
- fetal growth restriction
- breech presentation
- dysfunctional labor
- cesarean delivery

UTERINE DIDELPHYS (CLASS III) CLASS IV CLASS V


This anomaly is distinguished from bicornuate
and septate uteri by the presence of complete
nonfusion of the cervix and hemiuterine cavity.
Complications may include

- preterm delivery (20%)


- fetal growth restriction (10%)
- breech presentation (43%)
- cesarean delivery rate (82%)
BICORNUATE AND SEPTATE UTERI ACQUIRED REPRODUCTIVE TRACT ACQUIRED REPRODUCTIVE
(CLASSES IV AND V) ABNORMALITIES TRACT ABNORMALITIES
Vulvar Abnormalities
Marked increase in miscarriages Uterine Abnormalities
preterm delivery, abnormal fetal lie, and Bartholin Gland Lesions
Anteflexion
Due to diastasis recti and a pendulous abdomen
cesarean delivery. Needs no treatment during pregnancy.
If large enough to cause difficulty at delivery, - needle Management: abdominal binder.
aspiration Retroflexion
Abscess - broad-spectrum antimicrobials and drainage
uterus remains incarcerated in the hollow of the sacrum

abdominal discomfort and inability to void normally.


Condyloma Acuminata
Management:Knee chest position
Vaginal delivery unless so extensive that vaginal delivery may
be prohibited.

UTERINE ABNORMALITIES UTERINE ABNORMALITIES UTERINE ABNORMALITIES - MYOMA


Uterine Prolapse Leiomyoma
EFFECTS OF PREGNANCY ON MYOMA:
FIRST TRIMESTER-remained unchanged or

Degeneration during pregnancy: red or carneous increased in sizedue to estrogen.
Management: pessary in early pregnancy degeneration
Management during pregnany: myomectomy SECOND TRIMESTER-smaller myomas (2 to 6
cm) remained unchanged or increased in size,
Cystocele and Rectocele NOT DONE during pregnancy.Pain reliever given
for pain
whereas larger myomas became smaller

Cystocele or a rectocele can block normal fetal THIRD TRIMESTER- remained unchanged or
descent decreased, (estrogen receptor downregulation).
Management:Catheterization

UTERINE ABNORMALITIES - MYOMA UTERINE ABNORMALITIES - MYOMA OVARIAN ABNORMALITIES


Effects of Myoma in Pregnancy Management of Myomas during Pregnancy Mostly cystic.
preterm labor expectant management
Diagnosed by ultrasound or at cesarean
placental abruption Resection of myomas is generally delivery.
malpresentation contraindicated.
corpus luteal cysts,/ luteomas, which may
obstructed labor
be virilizing.
cesarean delivery
Ovarian hyperstimulation syndrome,-
postpartum hemorrhage
caused by ovulation-induction
Second to malignancy, the most serious
*** Two factors for morbidity are: myoma SIZE
and LOCATION complications are torsion or hemorrhage.
MANAGEMENT OVARIAN ABNORMALITIES QUESTIONS:
Recommendations:WHEN TO OPERATE 1.G1P0 PU 38 weeks in labor was
Resection of all cysts is recommended in:
All ovarian masses over 10 cm admitted.Uterine contractons occurred every 2
>10cm 6 to 10 cm with signs of malignancy by ultrasound minutes 60 secs duration.Cervix 2 cms dilated 1
Suspected rupture or torsion and MRI cm long.After 24 hours,cervix is still 3 cms
Give 17-OH-progesterone, 250 mg dilated 0.5 cms long.What is the best
Capable of obstructing labor intramuscularly weekly until 10 weeks if corpus management?
Cysts
5cm or less left alone, most luteum was removed a.oxytocin

undergo resolution b.sedation

Elective surgery at 16 to 20 weeks c.amniotomy

d.cesarean section

2.G1P0 39 weeks AOG admitted at 5 cms cervical 3.Failure in descent can be diagnosed if there is 4.Precipitatedelivery can be diagnosed in a
dilatation 0.5 cms long cephalic station no descent during which phase of labor? nulliparous patient if cervical dilatation is more
-1.Uterine contractions-200 montevideo a.latent than___cms/hr
units.Amniotomy done revealed clear AF.cervix a.2
b.acceleration
dilated to 6 cms after an hour,cephalic b.3
c.active
station-1.However after 3 hours cervix remained
d.deceleration c.4
at 6 cms.,cephalic station -1.What is the
diagnosis? d.5

a.prolonged active phase

b.protracted active phase

c.arrest in cervical dilatation

d.failure descent

5.Clinicalpelvimetry findings of a nulliparous 6.What plane of the pelvis is tested by theMueller 7.G1P0 38 weeks AOG has this leopolds findings:
patient revealed a prominent ischial Hillis Maneuver ? L1-breech L2-back on the right,small parts on
spines,convergent sidewalls,narrow sacrosciatic a.inlet the left,L3-cephalic L4 cephalic prominence on
notch.Which pelvic plane is contracted? b.midplane the right.On IE the mentum was directed at the
a.inlet sacrum.What is the manner of delivery?
c.outlet
b.midplane a.NSD

c.outlet b.forceps

c.vacuum

d.cesarean
8.A multipara in labor has this IE findings.The 9.A multipara was admitted in active labor.IE 10.What forceps is used to rotate a persistent
frontal sutures,anterior fontanel,orbital ridges revealed a gridiron feel with back down occiput transverse to anterior position?
and root of the nose are palpated.What is the position.What is the best management? a.simpsons
presentation? a.external cephalic version b.kiellands
a.sincipital b.internal podalic version c.pipers
b.brow c.low segment cesarean d.bartons
c.face d.classical cesarean
d.vertex

11.In shoulder dystocia ,the procedure of 12.External cephalic version to convert a breech 13.In partial breech extraction,the procedure of
hyperflexing the legs towards the abdomen is presentation to cephalic is recommended at what lateral deflection of the thigh,pressing on the
called weeks age of gestation? popliteal to flex the legs and deliver the foot is
a.pinards a.33 called
b.rubins b.35 a.loveset

c.mc roberts c.37 b.hibbard

d.zavanelli d.39 c.pinard

d.zavanelli

14.Which of the following structures is NOT 15.A G1P0 PU 12 weeks has a 15 cms 16.Which of the following is NOT used to deliver
derived from the mullerian duct? asymptomatic,ovarian cyst on the left an entrapped head in breech presentation?
a.uterus adnexa.What is the management? a.rubins maneuver

b.hymen a.expectant b.mauriceau smellie veit maneuver

c.upper third of the vagina b.immediate exploration c.suprapubic pressure

d.cervix c.explore at 16-20 weeks d.durshsen incision


d. explore after delivery
17.If there is no union of the mullerian duct ,the 18.A 17 year old consulted because of primary 19.19y/o G1P0 PU 34 weeks has painful myoma
abnormality produced is amenorrhea and cyclic pelvic pain.On uteri for 1 week.What is the degeneration of the
a.unicornuate uterus examination,bulging mass was noted at the myoma ?
introitus with no vaginal opening.What is the a.hyaline
b.bicornuate
diagnosis?
c.uterus didelphys b.carneous
a.endometrial polyp
d.septate uterus c.cystic
b.prolapsed myoma
d.sarcomatous
c.imperforate hymen

d.vaginal septum

ETIOLOGY OF MULTIPLE FETUSES


20.G3P2 PU 36 weeks came in fully dilated,frank Fraternal Twin
breech presentation ,station + 3.The attending Fertilization of two separate ova
physician waited for the spontaneous expulsion
of the breech up to the navel and assist the
MULTIFETAL GESTATION double ovum, or dizygotic

delivery with maneuvers from navel up to the


head.What is the described type of extraction? Identical Twins
a.Spontaneous breech delivery Twins arise from single fertilized ovum
DEPARTMENT OF OBSTETRICS AND GYNECOLOGY
b.total breech extraction FEU NRMF INSTITUTE OF MEDICINE single-ovum, monozygotic

c.complete breech extraction Have increased incidence of discordant

d.partial breech extraction


malformations

CAUSES: GENESIS OF DIZYGOTIC TWINNING


MONOZYGOTIC DIZYGOTIC
Superfetation
An interval longer than a menstrual cycle - relatively constant heredity
intervenes between fertilizations. - largely independent of increasing maternal age
race, heredity, age and increasing parity
Not yet proven in humans parity nutritional factors
Superfecundation assisted reproductive pituitary gonadotropin
Fertilization of two ova within the same therapy infertility therapy
menstrual cycle but not as the same coitus, nor
assisted reproductive
necessarily by sperm from the same male.
therapy
Genesis of Monozygotic
Twinning

MONOZYGOTI
C

CONJOINED TWIN

DETERMINATION OF ZYGOSITY
DETERMINATION OF ZYGOSITY DETERMINATION OF ZYGOSITY
Ultrasound the number of placenta Ultrasound the number of placenta
(chorionicity) can give a clue on zygosity (chorionicity) can give a clue on zygosity
Infant Sex and blood type Can determine chorionicity as early as the first Can determine chorionicity as early as the first
trimester trimester
Twins of the opposite sex are almost
Dichorionic: presence of two separate placentas and Dichorionic: presence of two separate placentas and
always dizygotic a thick generally 2mm or greater dividing a thick generally 2mm or greater dividing
membrane (twin peak sign) membrane (twin peak sign)
Infants of different blood types are Monochorionic: membrane generally less than 2mm Monochorionic: membrane generally less than 2mm
dizygotic in thickness and reveals only 2 layers. ( T sign) in thickness and reveals only 2 layers. ( T sign)
Placental Examination Placental Examination
Visual examination of the placenta and membranes Visual examination of the placenta and membranes
Placenta should be carefully delivered to preserve the Placenta should be carefully delivered to preserve the
attachment of the amnion and chorion to the placenta attachment of the amnion and chorion to the placenta
DETERMINATION OF ZYGOSITY
DIAGNOSIS OF MULTIPLE FETUSES DIAGNOSIS OF MULTIPLE FETUSES
Ultrasound the number of placenta DIFFERENTIAL DIAGNOSIS OF ENLARGED Ultrasonography
(chorionicity) can give a clue on zygosity FUNDIC HEIGHT Separate gestational
Can determine chorionicity as early as the first sacs
Multiple fetuses
trimester
Two fetal heads or two
Dichorionic: presence of two separate placentas and Elevation of the uterus by a distended bladder
a thick generally 2mm or greater dividing abdomens should be
Inaccurate menstrual history
membrane (twin peak sign) seen in the same plane
Hydramnios
Monochorionic: membrane generally less than 2mm
in thickness and reveals only 2 layers. ( T sign) Hydatidiform mole
Placental Examination Uterine myomas
Visual examination of the placenta and membranes
A closely attached adnexal mass
Placenta should be carefully delivered to preserve the
Fetal macrosomia (late in pregnancy)
attachment of the amnion and chorion to the placenta

DIAGNOSIS OF MULTIPLE FETUSES MATERNAL ADAPTATION PREGNANCY OUTCOME


Radiologic Examination Nausea and vomiting (severe) Abortion
Not useful
Maternal blood volume -greater 3x> than singleton pregnancies
Biochemical Test Cardiac output - 20% greater than Monozygotic : Dizygotic
BETA HCG- higher than singleton pregnancy, singleton
but not definite
Risk: 18:1
Blood loss via NSD 1000 ml

PREGNANCY OUTCOME PREGNANCY OUTCOME


Malformation
Low birthweight ( more on
- Defects from twinning itself. conjoined
twinning, acardiac anomaly, sirenomelia, neural- monozygotic than dizygotic)
tube defects, and holoprosencephaly.
Preterm birth
- Defects from vascular interchange
between monochorionic twins. microcephaly,
UNIQUE COMPLICATIONS
hydranencephaly, intestinal atresia, aplasia
cutis, or limb amputation.
- Defects as a result of crowding.
MONOAMNIONIC TWINS MONOAMNIONIC TWINS
Management CONJOINED TWINS
High risk for fetal death : -high fetal death
from cord entanglement
- cord entanglement ~ SIAMESE TWINS
- 1 hour daily FHR monitoring
- congenital anomaly beginning at
- preterm birth 26-28 weeks
- corticosteroid therapy- to
- twin to twin transfusion promote fetal lung maturity
sydrome - CS at 34 weeks

TYPES OF CONJOINED TWINS TYPES OF CONJOINED TWINS

TWIN TO TWIN TRANSFUSION


TWIN TO TWIN TRANSFUSION SYNDROME ACARDIAC TWIN
Blood is transfused from a donor twin to its DONOR TWIN- anemic and growth restricted Twin reversed-arterial-perfusion (TRAP) sequence is a
recipient sibling such that the donor becomes pale looking,oligohydramnios rare (1 in 35,000 births) but serious complication of
anemic and its growth may be restricted, monochorionic, monozygotic multiple gestation.
whereas the In the TRAP sequence, there is usually a normally
Recipient becomes polycythemic and may develop RECIPIENT TWIN-polycythemic,phletoric formed donor twin who has features of heart failure
circulatory overload manifest as hydrops. circulatory overload,heart as well as a recipient twin who lacks a heart
Donor twin - pale, recipient sibling plethoric failure due to (acardius) and various other structures.
Fetal consequences: Caused in the embryo by a large artery-to-artery
circulatory overload with heart failure placental shunt, often also accompanied by a vein-to-
hypervolemia,hyperbilirubinemia
vein shunt.
Occlusive thrombosis is also much more likely to
develop in this setting. The perfusion pressure of the donor twin overpowers
kernicterus,polyhydramnios that in the recipient twin, who thus receives reverse
Polycythemia may lead to severe
hyperbilirubinemia and kernicterus blood flow from its twin sibling.
TRAP SEQUENCE(TWIN REVERSED
ARTERIAL PERFUSION) MANAGEMENT DISCORDANT TWINS
Large artery to artery placental shunt Without treatment, the donor or "pump" twin has
Size inequality of twin fetuses
Vein to vein shunt been reported to die in 50 to 75 percent
Single shared placenta,the donor twin Methods of in utero treatment of acardiac As the weight difference within
twinning: goal is interruption of the vascular
overpowers the recipient twin.The recipient twin
communication between the donor and recipient
a twin pair increases, perinatal
utilizes the used arterial blood which perfuses
the lower body and disrupts growth of the upper twins. mortality increases
body
proportionately.
The earlier in pregnancy
DONOR- Heart Failure/RECIPIENT-Lacks heart discordancy develops, the more
serious the sequelae.

PATHOLOGY DIAGNOSIS OF DISCORDANCY MANAGEMENT


MONOCHORIONIC TWINS- placental vascular
anastomoses ULTRASOUND ASSESSMENT:ABDOMINAL
Ultrasonographic
DIZYGOTIC TWINS- different genetic growth
potential
CIRCUMFERENCE monitoring of growth
DISCORDANCY-WT(large twin)- Wt (smaller
twin) divided by weight of large twin Fetal surveillance

DISCORDANCY OF 25 to 30 % predicts adverse Delivery is usually not


outcome
performed for size
discordancy alone

DIET ANTEPARTUM SURVEILLANCE PREVENTION OF PRETERM DELIVERY


Assessment
of amnionic fluid volume- Bedrest
37-54
lb weight gain OLIGOHYDRAMNIOS
Tocolytictherapy
INCREASED CALORIC INTAKE-40-45 KCAL/ Corticosteroidsfor lung maturation
DAY The
nonstress test or biophysical profile
Cerclage NOT EFFECTIVE
IRON- 60 to 100 mg/day of iroN FETAL WELL BEING

Doppler evaluation of vascular resistance -


IUGR
DELIVERY OF TWIN FETUSES ANALGESIA AND ANESTHESIA MANAGEMENT
EPIDURAL ANALGESIA-Pain relief,internal 1.TWINS:
Complications of labor and delivery
podalic version,cesarean section
preterm labor . CEPHALIC CEPHALIC-VAGINAL/FORCEPS
uterine contractile dysfunction CEPHALIC/BREECH-VAGINAL DELIVERY/
BREECH EXTRACTION OF SECOND TWIN
abnormal presentation, prolapse of NON CEPHALIC FIRST TWIN- CESAREAN
the umbilical cord DELIVERY
premature separation of the placenta
2.TRIPLETS
OR HIGHER ORDER
immediate postpartum hemorrhage GESTATION::
CESAREAN DELIVERY

CLASSIFICATION OF HYPERTENSIVE DISORDERS


COMPLICATING PREGNANCY (WORKING GROUP OF THE NHBPEP
GESTATIONAL HYPERTENSION
HYPERTENSIVE 2000):

GestationalHypertension BP 140/90mmHg after 20 weeks AOG


DISORDERS IN
Preeclampsia
PREGNANCY Eclampsia
No proteinuria

Chronic Hypertension BP
returns to normal before12 weeks
postpartum transient hypertension
Superimposed Preeclampsia on
DEPARTMENT OF OBSTETRICS AND GYNECOLOGY
FEU-NRMF INSTITUTE OF MEDICINE Chronic hypertension

PREECLAMPSIA SEVERITY OF PREECLAMPSIA ECLAMPSIA


Increased certainty of
Minimum criteria preeclampsia
BP 140/90 mmHg BP 160/110mmHg after 20
weeks gestation
after 20 weeks Proteinuria 2.0 g/24 hours or
gestation
2 + dipstick
Serum creatinine > 1.2mg/dl
Proteinuria 300mg/ unless known to be previously Seizuresthat cannot be attributed to
elevated
24 hours or 1+ Platelets < 100,000/mm3 other causes in a woman with
dipstick Microangiopathic hemolysis
preeclampsia
(increased LDH)
Elevated ALT or AST
Persistent headache or other
cerebral or visual disturbances
Persistent epigastric pain
SUPERIMPOSED
PREECLAMPSIA
(ON CHRONIC
CHRONIC HYPERTENSION HYPERTENSION) RISK FACTORS
nulliparous women
New-onsetproteinuria 300 mg/24 hours in
race and ethnicity (African-American
hypertensive women with no proteinuria
BP140/90 mm Hg before pregnancy or ethnicity)
diagnosed before 20 weeks' gestation genetic predisposition
Asudden increase in proteinuria or blood
environmental factors
pressure or platelet count < 100,000/mm3 in
or hypertension first diagnosed after 20 chronic hypertension
women with chronic hypertension
weeks' gestation and persistent after 12
multifetal gestation
weeks' postpartum
maternal age over 35 years
obesity

ABNORMAL TROPHOBLASTIC INVASION


ETIOLOGY POTENTIAL CAUSES:
Exposed to chorionic villi for the first time. Abnormal trophoblastic invasion of
uterine vessels
Superabundanceof chorionic villi( twins or Maternal maladaptation to cardiovascular
hydatidiform mole) or inflammatory changes of normal
pregnancy
Preexisting vascular disease.
Immunological intolerance between
maternal and fetoplacental tissues
Genetics
Dietary deficiencies
Genetic influences

PATHOPHYSIOLOGY
PATHOGENESIS:

Vasospasm
Endothelial damage
HEMODYNAMIC CHANGES HELLP SYNDROME KIDNEY
Blood Volume
H Hemolysis ( LDH ) GFR
- hemoconcentration
(oliguria)
EL Elevated liver enzymes
uric acid
( AST or ALT)
Blood and Coagulation creatinine
- Thrombocytopenia results from platelet LP Low Platelet count Urine sodium
activation, aggregation, and consumption
Anatomical Changes: glomerular
An indication for capillary endothelial swelling
delivery PROTEINURIA

LIVER BRAIN UTEROPLACENTAL PERFUSION


headaches and visual symptoms
periportal hemorrhage gross hemorrhage due to ruptured arteries Compromised uteroplacental perfusion
principal postmortem lesions in eclampsia from vasospasm is the major
- edema culprit in increased perinatal morbidity
Subcapsular hematoma and liver
- hyperemia and mortality
rupture
- ischemia
- thrombosis
- hemorrhage
Blindness (amaurosis)

PREDICTORS OF PREGNANCY
INDUCED HYPERTENSION PREVENTION
Rollover test
Uric acid Dietary manipulation
Fibronectin Calcium supplementation
Coagulation activation Fish oil supplementation Delivery is the definitive
Oxidative stress
Cytokines
Antioxidant (Vit. C & E )
cure for preeclampsia
Low dose aspirin-PROVEN EFFECTIVE
Placental peptides
Fetal DNA
Uterine artery doppler velocimetry
MANAGEMENT IS BASED ON
MANAGEMENT severity of the disease and gestational age ECLAMPSIA
OBJECTIVES: of the fetus.
Preeclampsia complicated by generalized
tonicclonic convulsions.
Severe disease is delivered at 34 weeks.
1. Termination of pregnancy with the least
possible trauma to mother and fetus. Severe disease at < 34 weeks EXPECTANT provided
Differential Diagnosis:
2. Birth of an infant who subsequently thrives. maternal and fetal conditions are good.
Epilepsy
Mild disease or non-severe disease: await spontaneous
3. Complete restoration of health to the mother Encephalitis
labor but with more frequent check-ups
Meningitis
Cerebral tumor
Cysticercosis
Ruptured cerebral aneurysm

TOXICOLOGY OF MAGNESIUM ANTIHYPERTENSIVE MEDICATIONS


TREATMENT SULFATE
1. Control of convulsions using loading dose of Magnesium intoxication :WHAT TO MONITOR? Hydralazine-5mg every 15-20 mins
magnesium sulfate(iv), followed either by a - urine output is adequate Labetalol
continuous infusion or intramuscular
- the patellar or biceps reflex is present
- nonselective -blocker
-no respiratory depression
2. Intermittent intravenous or oral Nifedipine
Therapeutic level: 4 to 7 mEq/L
administration of an antihypertensive - 10-mg oral dose
Toxic Levels:
medication to lower blood pressure
- 10 mEq/L Patellar reflexes disappear Methyldopa-drug of choice for oral
- >10 mEq/L respiratory depression develops, medications
3. Avoidance of diuretics and limitation of
intravenous fluid administration. - >12 mEq/L respiratory paralysis and arrest Diuretics-for patients with pulmonary
4. Delivery. follow edema
AntIdote: Calcium gluconate, 1 g intravenously

ANALGESIA AND ANESTHESIA


DESIGN OF FORCEPS
Two crossing branches
four components:
Blade fenestrated or solid
Shank connects the handle and the blade
Epidural analgesia
Lock holds the forceps together
Handle to grip the forceps
two curves
cephalic curve conforms to the shape of the fetal
head
pelvic curve corresponds more or less to the axis of
the birth canal
Some varieties are fenestrated or pseudofenestrated to
permit a firmer hold on the fetal head
TYPES OF FORCEPS

SIMPSON FORCEPS
Most common forceps TUCKER-MCLANE KIELLAND FORCEPS
Parallel shanks
FORCEPS sliding lock,
blade is solid and the shank is minimal pelvic curvature,
English-style lock
narrow. and light weight
English lock
molded head (nulliparous) for deep transverse arrest
rounded head (multiparous)

FUNCTIONS OF OBSTETRIC FORCEPS


BARTON FORCEPS
PIPER FORCEPS 1. Traction
good forceps for
blade is similar to simpson The direction of the traction must be along the pelvic
shank is longer
rotation of head in
curvature
transverse arrest
it has a double pelvic curve for
aftercoming head in breech 2. Rotation
presentation
Carried out best in the mid pelvis

CLASSIFICATION OF FORCEPS
DELIVERIES
Procedure Criteria
INDICATIONS FOR FORCEPS CONTRAINDICATIONS TO USE FORCEPS
Outlet Scalp is visible at introitus without separating the labia Fetal indications Maternal indications
Fetal skull has reached pelvic floor
1. absence of proper indication
1. prolapsed of the 1. heart disease
Sagittal suture is in anteroposterior diameter or right or left 2. incompletely dilated cervix
occiput anterior or posterior position umbilical cord 2. hypertensive condition
3. marked cephalo-pelvic disproportion
Fetal head is at or on the perineum 2. premature separation 3. pulmonary injury or
Rotation does not exceed 45 degrees of the placenta compromise 4. unengaged fetal head
3. non-reassuring fetal
4. intrapartum infection 5. lack of experience on the part of the operator
Low 1. Leading point of fetal skull is at station +2 cm, and not on
5. neurologic condition
pelvic floor heart rate pattern
6. exhaustion
2. Rotation is 45 degrees or less (left or right occiput anterior to
occiput anterior, or left or right occiput posterior to occiput 7. prolonged second stage
posterior)
3. Rotation is greater than 45 degrees

Midpelvic Station above +2 cm but head is engaged


High Not included in classification
PREREQUISITES FOR FORCEPS
the head of the fetus is perfectly
APPLICATION PREPARATION FOR FORCEPS DELIVERY grasped only when the long axis of
1. Engaged head ANESTHESIA: pudendal block / regional analgesia the blades corresponds to the
or general anesthesia OCCIPITOMENTAL diameter
2. vertex or chin anterior
3. position must be precisely known bladder should be emptied
4. Fully dilated cervix
5. Ruptured membranes
6. no CPD

OUTLET FORCEPS DELIVERY


The forceps are applied as follows: For application of the right
blade, two or more fingers of
Two or more fingers of the right hand Horizontaltraction is
the left hand are introduced
are introduced inside the left exerted until the
into the right, posterior portion
posterior portion of the vulva and perineum begins to
of the vagina to serve as a
into the vagina beside the fetal head. bulge AND episiotomy
guide for the right blade, which
The handle of the left branch is then
may be performed if
is held in the right hand and
grasped between the thumb and two indicated
introduced into the vagina as
fingers of the left hand and the tip of described for the left blade.
the blade is gently passed into the After positioning, the branches
vagina between the fetal head and are articulated.
the palmar surface of the fingers of
the right hand

MORBIDITY FROM FORCEPS


OPERATIONS TRIAL FORCEPS
theoperator attempts delivery with the full
Forceps have been Maternal Morbidity Fetal Morbidity
knowledge that the vaginal delivery may not be
disarticulated and removed, 1. lacerations of the vulva, 1. poor apgar score
successful
and vagina and cervix and 2. cephalhematoma
MODIFIED RITGENS extension of episiotomy
3. caput
maneuver (arrow) is used to 2. urinary and rectal
4. facial mark or injury
complete delivery of the incontinence
head. 5. Erb palsy
3. infection
6. Fractured clavicle

7. Elevated bilirubin

8. Retinal hemorrhage
FAILED FORCEPS
VACUUM EXTRACTION
Vacuum Extraxtor Silastic cup vacuum
Failure of application: the forceps cannot be Causes
(Ventouse)
applied properly to the fetal head Disproportion

Malposition
metal cup or soft cup vacuum
Failure of Extraction: the forceps are applied Cervix not fully dilated extractors
but despite an all-out effort, delivery cannot be Constriction ring
accomplished high-pressure vacuum
Premature interference Silastic cup Mityvac
Mityvac instrument

INDICATIONS AND PREREQUISITES PREREQUISITES FOR VACUUM


Generally, the indications and prerequisites for the use of the vacuum extractor CONTRAINDICATIONS EXTRACTION
for delivery are the same as for forceps delivery
1. operator inexperience 1. head must be engaged
Fetal indications Maternal indications
2. inability to assess fetal position 2. fetus must present as a vertex or by the face
1. prolapsed of the umbilical
cord
1. heart disease with the chin anterior
3. high station
premature separation of 2. hypertensive
2.
the placenta condition 4. suspicion of cephalopelvic disproportion 3. position of the fetal head must be precisely
known
3. non-reassuring fetal heart
rate pattern
3. pulmonary injury or 5. face or other nonvertex presentations
compromise 4. cervix must be completely dilated
6. fetal coagulopathy
4. intrapartum 7. macrosomia 5. membranes must be ruptured
infection 6. no suspected cephalicpelvic disproportion
8. recent scalp blood sampling
5. neurologic condition
6. exhaustion
7. prolonged second
stage

TECHNIQUE COMPLICATIONS
scalp lacerations and bruising
proper cup placement full circumference of the cup should be palpated subgaleal hematomas
3 cm in front of the posterior fontanelle toward both before and after the vacuum and prior to cephalohematomas
the face traction. intracranial hemorrhage
traction should be intermittent and coordinated neonatal jaundice
Entrapment of maternal soft tissues- lacerations with maternal expulsive efforts subconjunctival hemorrhage
and hemorrhage and cup "pop-off." Abandon procedure if there are 3 pop offs clavicular fracture
shoulder dystocia
injury of sixth and seventh cranial nerves
Erb palsy
retinal hemorrhage
fetal death
CESAREAN DELIVERY AND
COMPARISON OF VACUUM EXTRACTION WITH PERIPARTUM
FORCEPS HYSTERECTOMY:
Vacuum Extraction
Cesarean birth of a fetus through incisions in the abdominal wall
Increase incidence of neonatal jaundice Delivery (laparotomy) and the uterine wall (hysterotomy).
Shoulder dystocia and cephalhematoma is doubled

Forceps Delivery
Higher frequency of maternal trauma and blood
Cesarean Hysterectomy that is performed at the time of cesarean
loss Hysterectomy delivery
More 3rd and 4th degree laceration

CESAREAN DELIVERY AND Postpartum


Hysterectomy
Hysterectomy that is done within a short time after vaginal
delivery
PERIPARTUM HYSTERECTOMY

INDICATIONS FOR CESAREAN DELIVERY CANDIDATES FOR VBAC INDICATIONS FOR PRIMARY CS
DYSTOCIA-most frequent indication
FETALDISTRESS
Prior cesarean delivery No more than 1 prior low-transverse CS delivery
BREECH PRESENTATION

Clinically adequate pelvis


Labor dystocia
No other uterine scars or previous uterine rupture
Fetal distress
Physician immediately available throughout active labor
who is capable of monitoring labor and performing
Breech presentation emergency CS

Availability of anesthesia and personnel for emergency CS

TECHNIQUE FOR CESAREAN


DELIVERY TECHNIQUE FOR
TECHNIQUE FOR
ABDOMINAL INCISIONS CESAREAN DELIVERY CESAREAN DELIVERY
ABDOMINAL INCISION
ABDOMINAL INCISION TRANSVERSE Modified Pfannenstiel incision
midline vertical or a VERTICA Infraumbilical midline
INCISION

suprapubic transverse incision L


INCISION
vertical incision

ADVANTAGE Cosmetic
Stronger incision
paramedian or midtransverse Less dehiscence or hernia

incision-RARELY Advantage Can be rapidly extended


DISADVANTAGE Suboptimal exposure
S Reentry is more difficult due to scarring
MAYLARD rectus muscle divided w/ scissors and scalpel
Quickest to make INCISION useful in women with scarring resulting from
previous Pfannensteil incision
TECHNIQUE FOR CESAREAN DELIVERY TECHNIQUE FOR CESAREAN DELIVERY TECHNIQUE FOR CESAREAN DELIVERY
UTERINE INCISION
UTERINE INCISION
UTERINE INCISION CLASSICAL Vertical incision in the body of the uterus
INCISION
KERR lower uterine segment
transverse incision KRONIG Low segment vertical Indications LOWER SEGMENT CANT BE EXPOSED
incision 1. bladder adhesions
Operation of choice 2. myoma
3. invasive CA of the cervix
Advantages easier to repair
Disadvantage extensive dissection of the
less incidence of rupture s bladder Transverse lie of large fetus
Placenta previa with anterior implantation
May extend to the cervix
and bladder Very small fetuses with lower uterine segment not
LESS adherence to bowel
thinned out
or omentum
More likely to rupture massive maternal obesity

POSTMORTEM CESAREAN
DELIVERY PERIPARTUM HYSTERECTOMY PERIPARTUM HYSTERECTOMY
INDICATION uterine atony (most common )
S
MAJOR Increased blood
cesarean delivery Lower uterine segment bleeding
COMPLICATIO loss
that is performed Laceration of major uterine vessels
NS Possibility of
urinary tract
on a woman who Placenta accreta
Large myoma damage
has just died Severe cervical dysplasia or CIS
Increased morbidity associated
with emergency hysterectomy
Intrauterine infection
Grossly defective scar

QUESTIONS:
1.A G1P1 PU 13 weeks has an ultrasound result 2.Which of the following must NOT be done in a 3.Which of the following characterizes the
twin pregnancy with single chorion and 2 monoamnionic monochorionic twins? recipient in twin to twin transfusion//
amnion.When does the division of the a.Daily CTG at starting at viability a.anemic
monozygotic twin occurred ? b.steroids at 26-28 weeks b.hyperbiliribunemia
a.0-4 days
c.Deliver at 38 weeks c.IUGR
b.4-8 days
d.terminate by CS d.oligohydramnios
c.8-12 days

d.>13 days
4.Which of the following presentations in 5.21y/o G1P0 PU 32 weeks cephalic,complaining 6.36 y/o G1P0 PU 36 weeks was admitted
multifetal pregnancy can be delivered vaginally of headache.BP-160/100 mmhg.Urine protein ++ because of blurring of vision.BP-150/100
in multiparous patient? +.What is the diagnosis? mmhg,urine protein +++.Lab tests revealed low
a.twin breech-cephalic a.gestational hypertension platelets,increased LDH,SGPT and alkaline
phosphatase.What is the complete diagnosis?
b.twin-cephalic breech b.chronic hypertension
a.Pre eclampsia non severe
c.twin-cephalic-transverse c.transient hypertension
b.Pre eclampsia,severe
d.triplets all cephalic d.severe pre eclampsia
c.Pre eclampsia,severe, HELLP syndrome

d.Pre eclampsia ,severe,DIC

7.Which of the following is the most effective in 8.G2P 0 PU 35 weeks complained of epigastric 9.Which forcep is described to have a longer
the prevention of pre eclampsia? pain .BP-190/100 mmhg.Lab test revealed low shank and a double pelvic curve?
a.low dose aspirin platelets and increased LDH.What is the a.bartons
definitive management of this patient?
b.high dose calcium b.pipers
a.control hypertension with hydralazine
c.fish oil c.simpsons
b.prevent convulsion with MG SO4
d.antioxidants d.kiellands
c.weekly surveillance testing

d.terminate pregnancy

10.In what diameter of the pelvis will the forcep 11.How many pop offs during vacuum extraction 12.Which of the following will qualify a patient
fits during application? before you will abandon the procedure? for a vaginal birth after a cesarean section?
a.biparietal a.1 a.one previous Classical CS

b.occipitofrontal b.2 b.no previous uterine rupture in last 2yrs

c.occipitomental c.3 c.can be performed in a lying in with physician

d.suboccipitobregmatic d.4 available


d.The obstetrician and anesthesiologist must be
available
13.What is the MOST frequent indication for 14.Which of the following is a disadvantage of 15.What is the most frequent indication for CS
primary CS? pfannesteil incision? hysterectomy?
a.malpresentation a.weak a.atony

b.dystocia b.more dehiscence b.laceration of uterine vessels

c.fetal distress c.difficult re entry c.accreta

d.maternal illness d.faulty healing d.myoma

PUERPERAL INFECTION PUERPERAL FEVER


any bacterial infection of the genital tract after temperature 38.0 C (100.4 F) or higher at any
delivery 2 of the first 10 days postpartum, exclusive of the
first 24 hours and to be taken by mouth by a
standard technique at least 4 times daily

PUERPERAL INFECTION

DIFFERENTIAL DIAGNOSIS
OF PUERPERAL FEVER GENITAL TRACT INFECTION
PUERPERAL FEVER
Incidence : Genital Tract Infection Risk Factors :
Puerperal Sepsis : 32.1% Extragenital Causes prolonged rupture of membranes

Intrapartum Sepsis (Chorioamnionitis) : 2.1% Respiratory Complications intrauterine electronic monitoring

Postabortal Sepsis : 1.5%


Pyelonephritis extensive vaginal / uterine manipulation
Breast Engorgement
Superficial or Deep Venous Thrombophlebitis
Incisional Wound Abscess
RESPIRATORY COMPLICATIONS
PYELONEPHRITIS BREAST ENGORGEMENT
most often seen within the first 24 hours ,most bacteriuria, pyuria, costovertebral brief temperature elevation rarely
often seen in women delivered by cesarean
section angle tenderness and spiking exceeds 39.0 C and no longer than
atelectasis, aspiration pneumonia or occasionally temperature 24 hours
bacterial pneumonia
Atelectasis is prevented by routine coughing, early
ambulation, deep breathing every 4 hours for 24 nausea and vomiting developing Bacterial Mastitis- fever develops
hours
later later,persistent and with signs and
symptoms of breast infection
Pyuria and bacteriuria

SUPERFICIAL OR DEEP VENOUS


THROMBOPHLEBITIS POSTPARTUM UTERINE INFECTION PREDISPOSING FACTORS

minor temperature elevations, endometritis, endomyometritis, Route of delivery


Labor
painful, swollen leg endoparametritis Rupture of membranes
calf tenderness or occasionally the route of delivery is the single Number of vaginal examinations

femoral triangle area tenderness most significant risk factor for Low socioeconomic status
Others
the development of postpartum

treated with intravenous Anemia


uterine infection Nutrition
heparin Sexual intercourse

ROUTE OF DELIVERY LABOR RUPTURE OF MEMBRANES


Vaginal Delivery 2-3% prolonged duration of labor = increased cervical if > 6 hours = (+) pathogenic bacteria
Risk factors and vaginal examinations 95% developed endometritis
prolonged rupture of membranes and labor postpartum morbidity after cesarean section
multiple cervical examination
internal fetal monitoring
Intra amniotic infection
Cesarean Delivery 5-6%
NUMBER OF VAGINAL EXAMINATION LOW SOCIOECONOMIC STATUS OTHERS
increased postpartum endometritis possible
causes : differences in vaginal flora, Anemia
hygiene and nutrition does not predispose to infection but normal
hemoglobin may prevent infection by increasing
transferrin(with significant antibacterial function)
Nutrition
cell mediated immunity is impaired in
malnourished
Sexual Intercourse
Not clearly demonstrated

BACTERIA COMMONLY RESPONSIBLE


FOR FEMALE GENITAL INFECTIONS METRITIS
Aerobes

polymicrobial

Group A, B, and D streptococci


Enterococcus
Gram-negative bacteriaEscherichia coli, Klebsiella, and Proteus species
Pathogenesis :

Staphylococcus aureus
Staphylococcus epidermidis involves the placental implantation site and the
Gardnerella vaginalis deciduas and adjacent myometrium
Anaerobes
Peptococcus species Foul, profuse, bloody and frothy discharge
Peptostreptococcus species Leukocytic infiltration
Bacteroides fragilis group
Prevotella species
Clostridium species
Fusobacterium species
Mobiluncus species
Other
Mycoplasma species
Chlamydia trachomatis
Neisseria gonorrhoeae

ANTIMICROBIAL REGIMENS FOR PELVIC


INFECTION FOLLOWING CESAREAN DELIVERY
WOUND INFECTIONS NECROTIZING FASCIITIS
Regimen Comments
Clindamycin 900 mg +gentamicin 1.5 "Gold standard," 9097% efficacy, Risk factors: rare but fatal
mg/kg, q8h intravenously once-daily gentamicin dosing obesity involve abdominal incisions following cesarean
acceptable diabetes delivery, or may complicate episiotomy or
plus ampicillin Added to regimen with sepsis perineal laceration
corticosteroid therapy
syndrome or suspected enterococcal significant tissue necrosis
immunosuppression
infection
anemia
Clindamycin+aztreonam Gentamicin substitute with renal
insufficiency poor hemostasis with hematoma formation

Extended-spectrum penicillins Piperacillin, ampicillin/sulbactam

Extended-spectrum cephalosporins Cefotetan, cefoxitin, cefotaxime

Imipenem + cilastatin Reserved for special indications


NECROTIZING FASCIITIS NECROTIZING FASCIITIS NECROTIZING FASCIITIS
Risk factors for fasciitis Treatment
diabetes, obesity, and hypertension Clindamycin given with a beta-lactam treatment : wide debridement of all infected
may be caused by a single virulent bacterial antimicrobial - most effective regimen tissue, split thickness skin grafts
species such as group A beta-hemolytic promptly debriding wide margins of the fascial mortality -50% even if aggressive excision is
streptococcus, but more commonly they are incision performed
polymicrobial

TOXIC SHOCK SYNDROME TOXIC SHOCK SYNDROME


Staphylococcus aureus toxic shock syndrome
acute febrile illness with severe multisystem toxin 1
derangement
fever, headache, mental confusion, diffuse Therapy
: SUPPORTIVE, anti-staphylococcal
macular erythematous rash, subcutaneous antimicrobials, massive fluid replacement,
edema, nausea, vomiting, watery diarrhea, mechanical ventilation with PEEP, renal dialysis
marked hemoconcentration CARDIOVASCULAR DISEASE
renal failure hepatic failure, DIC
circulatory collapse

HEART DISEASE IN PREGNANCY


There are certain peaks of cardiac activity during DIAGNOSTIC STUDIES
pregnancy:
ECHOCARDIOGRAPHY
Early third CHEST XRAY
trimester
ELECTROCARDIOGRAM

During Durin
puer- g
perium labor

During
delivery
It is during these periods when cardiac failure is likely to occur.
CLINICAL CLASSIFICATION CLINICAL CLASSIFICATION MANAGEMENT OF CLASS I & II

Slight limitation Vaginal


of physical Marked
Uncompromised activity: limitation of Severely
delivery
(no limitation of These women are physical activity: Compromised
physical comfortable at The women are (inability to perform
activity): rest, but if comfortable at any physical
Class These women do Class ordinary physical
activity is
Class rest, but less than Class activity without
discomfort):
Mode of
I not have symtoms
of cardiac
II undertaken,
discomfort results
III ordinary activity
causes excessive
IV Symptoms of cardiac
insufficiency or angina
Delivery:
insuffuciency or in the from of fatigue, even at rest, and if any
experience anginal excessive fatigue, palpitation, activity is undertaken, epidural
pain palpitation, dysnea, or anginal discomfort is increased Relief anesthesia
dyspnea, or pain from GA
anginal pain
pain: avoid
subarachnoid block

MANAGEMENT OF CLASS III AND IV


FACTORS THAT WILL CAUSE HEART
DISEASE
FAILURE IN THE PUERPERAL PERIOD CONTRACEPTION

Anemia Progestin only pills


OCP-causes thromboembolism
Hemorrhage IUD-contraindicated /causes subacute bacterial qsevere cardiac disease should consider
endocarditis pregnancy interruption.
Infection Tubal Sterilization-delayed until patient can q prolonged hospitalization or bed rest
ambulate qEpidural analgesia
thromboembolism qVaginal delivery (Forceps/vacuum)
qCesarean delivery is limited to obstetrical
indications.

ANTICOAGULATION OF PREGNANT
ANTIMICROBIAL PROPHYLAXIS
WOMEN WITH CARDIAC DISORDERS
TO BE GIVEN 30-60 MINUTES PRIOR TO DELIVERY usually for patients with mechanical prosthetic
Ampicillin 2gms IV or Amoxicillin 2 gms oral valves

If penicillin sensitive: Unfractionated Heparin


Cefazolin or Ceftriaxone 1 gm IV given at 6-12 weeks. Then
If with history of anaphylaxis: resumed at 36 weeks and discontinued before
delivery
Clindamycin 600 mg IV PULMONARY DISORDERS
Warfarin
If with enterococcal infection:
started at 13 weeks and discontinued at 36 weeks
add vancomycin resumed postpartum
PATHOPHYSIOLOGY
ASTHMA ASTHMA

seenfrequently during
The hallmarks of asthma are: Because F-series
pregnancy qreversible airway obstruction
chronic inflammatory
prostaglandins and
tenacious mucus, & mucosal
ergonovine exacerbate
airway disorder with a edema
qairway inflammation asthma, should be
major hereditary avoided if possible
component.

EFFECTS OF PREGNANCY ON ASTHMA EFFECTS OF ASTHMA ON


PREGNANCY TREATMENT
Treatment depends on the severity of
There is no Unless there is disease

evidence that severe disease, agonists help to abate


pregnancy has a pregnancy outcomes bronchospasm,
predictable effect on are generally corticosteroids treat
the inflammatory
underlying asthma. excellent component

REGIMENS RECOMMENDED FOR LABOR AND DELIVERY


MANAGEMENT OF ACUTE ASTHMA OUTPATIENT MANAGEMENT
1st line therapy: Beta Adrenergic agonists
qMaintenance medications are
continued through delivery
MOA: Bronchial smooth muscle
relaxation qFor mild asthma, inhaled -
100 mg of hydrocortisone given
agonists intravenously every 8 hours during
Terbutaline Epinephrine q For persistent asthma, inhaled labor and for 24 hours after
Albuterol Isoproterenol corticosteroids delivery
Isoetharine Metaproterenol
INCIDENCE AND CAUSES
LABOR AND DELIVERY PNEUMONIA
Epidural is ideal
Pregnancy itself does not
predispose to pneumonia
Avoid tracheal intubation Any pregnant woman
suspected of having
S.pneumoniae is the most
pneumonia should undergo
Oxytocin and PGE2 ok chest radiography common cause

PGF2 and ergotamine derivatives


NOT OK

CHEST RADIOGRAPHS IN A PREGNANT WOMAN WITH RIGHT


LOWER LOBE PNEUMONIA. A. COMPLETE OPACIFICATION OF THE
RIGHT LOWER LOBE (ARROWS) IS CONSISTENT WITH THE
CLINICAL SUSPICION OF PNEUMONIA. B. OPACIFICATION
DIAGNOSIS (ARROWS) IS ALSO SEEN ON THE LATERAL PROJECTION.
MANAGEMENT

Antimicrobial treatment is
qSymptoms : cough, dyspnea, empirical
sputum production, and
pleuritic chest pain. macrolideazithromycin,
qChest radiography is essential clarithromycin, or erythromycin.
for diagnosis

PREVENTION INFLUENZA PNEUMONIA


PREGNANCY OUTCOME WITH PNEUMONIA

Maternal Mortality: 0.8% qRNA viruses: Influenza A and B


qPneumococcal vaccine is 60- to 70-
infection is self-limited.
percent protective
Preterm labor qVaccine is not recommended for
healthy pregnant women Pneumonia is the most common
q Recommended for complication, and it is difficult to
PPROM
immunocompromised patients distinguish from bacterial
pneumonia.
CHEST RADIOGRAPH OF A 30-WEEK
PREGNANT WOMAN WITH CONFIRMED
MANAGEMENT
INFLUENZA A PNEUMONIA. MANAGEMENT
Supportivetreatment with antipyretics and
bed rest is recommended for uncomplicated Oseltamivir is given orally, 75
influenza.
mg twice daily, or zanamivir is
given by inhalation, 10 mg twice
daily

PREVENTION
TUBERCULOSIS DIAGNOSIS OF PTB

MOT: inhalation of Mycobacterium Chest X-ray:


Vaccination for tuberculosis; a granulomatous
infiltrative pattern
influenza A is pulmonary reaction
cavitation
recommended Clinical manifestations: mediastinal lymphadenopathy
during pregnancy cough with minimal sputum production
low-grade fever
hemoptysis AFB Culture
weight loss AFB Smear

TUBERCULOSIS AND PREGNANCY TUBERCULOSIS SCREENING TUBERCULOSIS


Effect on Pregnancy
Purifiedprotein
Latent
preterm delivery

low birthweight derivative (PPD)


growth-restricted infants
If negative (<5mm),no
Active
perinatal mortality

further evaluation is needed


Without antituberculosis therapy,
pregnancy likely has adverse If positive, (>5mm) CXR
effects on the course of active
tuberculosis
ACTIVE INFECTION
Drug regimen: Isoniazid, Rifampin, and ACTIVE INFECTION NEONATAL TUBERCULOSIS
Ethambutol
Isoniazid resistant tuberculosis - Pyrazinamide Congenital Tuberculosis
is added to initial regimen, given for 9 months aminoglycosides bacilli blood placenta
Breastfeeding is not prohibited during streptomycin, kanamycin, aspiration of infected secretion
antituberculosis therapy
Pyridoxine, 25mg/day orally, to decrease hepatic
amikacin, and capreomycin during delivery
toxicity due to isoniazid are ototoxic to the fetus
and are contraindicated

NEONATAL TUBERCULOSIS RENAL DISEASES DURING ASSESMENT OF RENAL DISEASE IN PREGNANCY


PREGNANCY
Urinalysis
Ultrasound

Neonatal infection is unlikely if the Intravenous Pyelography


mother with active disease has been Cystoscopy
treated before delivery
MRI of renal masses
Renal biopsy ( usually postponed
until pregnancy is completed)

URINARY TRACT INFECTIONS Table 481. Antimicrobial Agents Used for Treatment of Pregnant Women with Asymptomatic
Bacteriuria
ASYMPTOMATIC BACTERIURIA Single-dose treatment
Amoxicillin, 3 g
v Most common bacterial infections in v persistent, actively multiplying
Ampicillin, 2 g
pregnancy bacteria in women who have no
Cephalosporin, 2 g
v 90 percent are caused by Escherichia coli symptoms
Nitrofurantoin, 200 mg
Trimethoprim-sulfamethoxazole, 320/1600 mg
valso more common in diabetics
3-day course
DIAGNOSIS: A clean-voided Amoxicillin, 500 mg three times daily
specimen containing more than
Ampicillin, 250 mg four times daily
100,000 organisms per mL
Cephalosporin, 250 mg four times daily
Ciprofloxacin, 250 mg twice daily
Levofloxacin, 250 mg daily
Nitrofurantoin, 50 to 100 mg four times daily; 100 mg twice daily
Trimethoprim-sulfamethoxazole, 160/800 mg two times daily
CYSTITIS & URETHRITIS
ACUTE PYELONEPHRITIS ACUTE PYELONEPHRITIS
v is the most common serious medical
v Chlamydia trachomatis, Frequency, complication of pregnancy . v Differentialdiagnosis: labor,
urgency, dysuria, and sterile pyuria , urine chorioamnionitis, appendicitis, placental
culture with no growth. v urosepsis was the leading cause of septic abruption or infarcted myoma
v Management: AZITHROMYCIN shock v Monitor plasma creatinine

v more common in the second trimester ,


nullipara and young age

ANTIMICROBIAL THERAPY FOR ACUTE


ACUTE PYELONEPHRITIS PYELONEPHRITIS
Table 482. Management of the Pregnant Woman with Acute Pyelonephritis

CLINICAL FINDINGS: 1. Hospitalization v Ampicillin plus gentamicin


2. Urine and blood cultures

v anorexia, nausea, and vomiting. 3. Hemogram, serum creatinine, and electrolytes v Cefazolin or ceftriaxone
v Fever as high as 40 C 4. Monitor vital signs frequently, including urinary output; consider indwelling
catheter
v Tenderness at the costovertebral v Extended spectrum antibiotic
5. Intravenous crystalloid to establish urinary output to >/=50 mL/hr-cornerstone***
angles. v Cephalosporins & gentamicin
6. Intravenous antimicrobial therapy
v leucocytosis some in clumps. excellent vs E. Coli
7. Chest radiograph if there is dyspnea or tachypnea
v Oral therapy after discharge for
8. Repeat hematology and chemistry studies in 48 hours
7 to 14 days
9. Change to oral antimicrobials when afebrile
10. Discharge when afebrile 24 hours; consider antimicrobial therapy for 7 to 10 days
11. Urine culture 1 to 2 weeks after antimicrobial therapy completed

THYROID FUNCTION CHANGES DURING DIAGNOSTIC SIGNS OF


PREGNANCY: HYPERTHYROIDISM/ THYROTOXICOSIS:
ENDOCRINE DISORDERS IN thyroid binding globulin
PREGNANCY Severe tachycardia
daily thyroxine (T4) and T3
elevated sleeping pulse rate
thyromegaly
DEPARTMENT OF OBSTETRICS AND GYNECOLOGY Thyroid Function Tests to exophthalmos
Far Eastern University Nicanor Reyes Medical Foundation request during pregnancy failure to gain weight
serum free T4 and
- TSH, free T4, free T3

TSH
HYPERTHYROIDISM / THYROTOXICOSIS TREATMENT:
AND PREGNANCY MEDICAL
Thioamide drugs:

1. propylthiouracil-preferred because it Ablation


with radioactive iodine is
Graves Disease
partially inhibits the conversion of T4 to CONTRAINDICATED during pregnancy
- an autoimmune process due to thyroid
stimulating antibodies. T3 and it crosses the placenta less
readily. Maternal
thyroidectomy NOT DONE
DUE TO BLEEDING
2.methimazole.

S/E of methimazole:
esophageal atresia
choanal atresia
aplasia cutis

HYPERTHYROIDISM / THYROTOXICOSIS HYPERTHYROIDISM / THYROTOXICOSIS


AND PREGNANCY AND PREGNANCY FETAL AND NEONATAL EFFECTS
Thyroid Storm and Heart Failure
Thyroid Storm and Heart Failure goitrous thyrotoxicosis caused by
Management:
placental transfer of thyroid stimulating
- propylthiouracil
Thyroid Storm an acute, life-
antibodies or goitrous hypothyroidism due
- iodide (inhibits thyroidal release of T3/T4)
threatening, to thioamides treatment(less common)
IV, oral (SSKI), Lugol solution
hypermetabolic state - Lithium carbonate ( if with allergy to iodide)
- Dexamethasone ( to further inhibit
Heart Failure due to cardiomyopathy peripheral conversion of T4 to T3) Hydrops and fetal demise
- B-blocker (propranolol) to control
tachycardia

HYPOTHYROIDISM HYPOTHYROIDISM HYPOTHYROIDISM


Due
to glandular destruction by associated with infertility EFFECTS IN FETUS & INFANTS:
autoantibodies
- neuropsychological development.
high incidence of preeclampsia, placental
Overt
hypothyroidism: abruption low birth weight & stillborn
impaired psychomotor development.
TSH, FT3 and FT4 infants.
PREGNANCY INDUCED STATE OF PERIPHERAL
HYPOTHYROIDISM RESISTANCE TO INSULINDUE TO:

MANAGEMENT: Thyroxine replacement GESTATIONAL Increasedinsulin response to


glucose ( increase plasma level &
Iodine Deficiency: DIABETES MELLITUS duration)
RDA: 220 ug / day
Mild deficiency: intelectual impairment Reduced peripheral uptake of
Severe deficiency: endemic cretinism glucose ( increased plasma level
duration)
Congenital Hypothyroidism:
Detection is part of newborn screening Suppressed glucagon response.

SCREENING:
FETAL EFFECTS:
screening
for gestational diabetes - performed MATERNAL EFFECTS: Miscarriage
between 24 and 28 weeks Preterm delivery
Preeclampsia AlteredFetal growth
a50-g oral glucose challenge test is followed by a bacterial infection Fetal malformation
diagnostic 100-g oral glucose tolerance test if
Macrosomic fetus Hydramnios
result IS MORE THAN 140 mg /dl
Hydramnios

Increase maternal mortality

NEONATAL MORTALITY AND MORBIDITY


Respiratory Distress Syndrome Hyperbilirubinemia and Polycythemia MANAGEMENT:
Delayed fetal lung maturation Cardiomyopathy Insulin therapy - when standard dietary
Hypoglycemia Hypertrophic cardiomyopathy management does not maintain the:
hyperplasia of the fetal -islet cells induced by chronic Long-Term Cognitive Development FBS- less than 95 mg/dL or the 2-hour
maternal hyperglycemia. postprandial less than 120 mg/dL.
Inheritance of Diabetes
Hypocalcemia exercise
POSTPARTUM SURVEILLANCE OF
GDM PATIENTS:
75-g oral glucose tolerance test at 6 to 12 MATERNAL EFFECTS OF OVERT DIABETES: FETAL EFFECTS OF OVERT DIABETES
weeks after delivery.
Diabetes nephropathy
Perinatallosses
Diabetic retinopathy
FetalMacrosomia
Diabetic neuropathy
Low dose OCP SAFE FOR GDM PATIENTS Abortion
Preeclampsia
Ketoacidosis Preterm delivery at 34 weeks

Infection Malformations

Unexpected fetal demise


Hydramnios

MANAGEMENT OF OVERT MANAGEMENT OF OVERT


DIABETES: DIABETES:
The American Diabetes Association has defined Hospitalization if with hypertension.
optimal preconceptional glucose control using - Maternal Alpha feto protein 16-20 weeks Fetal wellbeing tests.
insulin: FBS-70 to 100 mg/dL :postprandial NST beginning at 28 30 weeks
- Congenital anomaly screening at 18-20 weeks.
values of less than 140 mg/dL :less than 120 mg/ If non reactive do CST or BPS
dL at 1 and 2 hours -Ultrasound later in pregnancy for macrosomia

or IUGR If clinical conditions deteriorates as in preeclampsia


or rapidly developing polyhydramnios deliver even
Folate 400ug/ day TO decrease the risk of neural if with reassuring test.
tube defect.

QUESTIONS:
CONTRACEPTION: 1.36y/o G3P2 PU 33 weeks has PPROM for 8 2.What is the most important factor for the
hours.She delivered after 24 hours of labor.On development of genital tract infection during
the third postpartum day she developed vaginal puerperium ?
Estrogen Progesterone contraindicated if with
vascular involvement bleeding,fever and hypogastric pain.Cervix a.number of cervical examination
tender on wriggling,uterus enlarged to 5 months
b.route of delivery
size and tender.What is the diagnosis?
Progestin only & implants c.length of labor
a.cystitis
IUD d.anemia
b.endometritis
Barrier method c.pyelonephritis
Sterilization
d.thrombophlebitis

3.34y/o G3P3 post CS for 1 week due to prolonged 4.What is the microorganism implicated in Toxic 5.25y/oG1P0 PU 12 weeks with RHD is
labor complained of vaginal bleeding,abdominal Shock syndrome? comfortable at rest but complains of dyspnea
pain and foul smelling discharge.What is the a.staphylococcus aureus while washing the dishes or even when brushing
BEST antibiotic management ? her teeth.What is the new York classification of
b.streptococcus pyogenes
a.ampicillin and gentamycin this patient?
c.Escherichia Coli
b.broad spectrum cephalosporin a.1
d.Pseudomonas
c.clindamycin and gentamycin b.II

d.meropenem c.III

d.IV

6.What is the best mode of Delivery for a 21y/o 7.A G3P3 asthmatic patient delivered to a live 8.Which of the following anti TB medications is
G1P0 with RHD mitral stenosis? baby .Which of the following should NOT be contraindicated during pregnancy?
a.NSD under sedation given postpartum? a.streptomycin

b.assisted vaginal under pudendal a.antibiotics b.rifampicin

c.forceps extraction under epidural b.hydrocortisone c.pyrazinamide

d.cesarean section c.terbutaline d.ethambutol


d.ergonovine

9.23 y/o G4P1 PU 21 weeks has an asymptomatic 10.32y/oG2P1 PU 35 weeks has recurrent UTI 11.What is/ are the laboratory tests needed to
UTI.Urinalysis showed plenty of pus cells and complains of fever,upper back pain,nausea evaluate a patient with thyroid disease?
however Urine culture is negative.What is the and vomiting.What is the cornerstone in the a.MRI
microorganism implicated? management of this patient ? b.thyroid ultrasound
a.E. Coli a.request for creatinine
c.TSH ,FT3FT4
b.chlamydia b.empiric antibiotics
d.thyroid scan
c.pseudomonas c.hydration with IVF

d.bacterial vaginosis d.antipyretic


12.23y/o G1P0 PU 16 weeks has diffuse thyroid 13.When is the recommended age of gestation to 14.21 y/o G1P0 has a result of 145 gms/dl in the
enlargement with exopthalmos.TSH is low while screen for gestational DM based on American 50 gms OGCT.What is the next management for
FT4 is elevated.What is the BEST treatment for College of OB GYN? this patient?
this patient? a.first trimester a.start oral hypoglycemics
a.propanolol b.16-20 weeks b.start insulin
b.iodine c.24-28 weeks c.Do 100 gms OGTT
c.prophylthiuracil d.30-34 weeks d.manage as normal pregnancy
d.thyroxine

15.Which of the following is NOT recommended 16.Which of the following vaccines must be given
in patients with Overt DM? to all pregnant patient?
a.alpha feto protein at 16-20 weeks a.hepatitis A

b.congenital scan at 18-20 weeks b.HPV

c.weekly doppler velocimetry c.influenza


Williams Obstetrics 24th Edition
d.regular ultrasound for growth d.pneumonia

INFECTIOUS DISEASES

First
Half of Pregnancy
CONGENITAL FETAL AND NEONATAL
VARICELLA
VARICELLA ZOSTER VIRUS SYNDROME INFECTION VARICELLA ZOSTER VIRUS
HERPES ZOSTER OR SHINGLES Chorioretinitis, micropthalmia,
Reactivation of varicella Infection cerebral cortical atrophy, growth
restriction, hydronephrosis, limb Exposure Risk Based on Age of Gestation
Unilateral dermatomal vesicular eruption with
hypoplasia, cicatricial skin lesion < 13 weeks 0.4%
severe pain
13-20 weeks Highest risk
>20 weeks None
Before or During Delivery: Disseminated
visceral, CNS disease (FATAL)
5 DAYS BEFORE AND 2 DAYS AFTER DELIVERY
Recommendation
for exposure: Give
VARICELLA ZOSTER Immunoglobulin
VARICELLA ZOSTER VIRUS VARICELLA ZOSTER VIRUS VARICELLA ZOSTER VIRUS

DIAGNOSIS: MANAGEMENT VACCINATION:

MATERNAL VIRAL EXPOSURE:


Clinical Manifestation
VARIZIG ( Varicella Zoster Immunoglobulin) NOT recommended for pregnant OR within a
Tzanck Smear month following each vaccine dose
within 96 hours of exposure up to 10 days
Tissue culture Not secreted in breastmilk
MATERNAL INFECTION:
Direct Fluorescent Antibody Testing Isolation
NAAT (Nucleic Acid Amplification Test )of Supportive
Amniotic Fluid Chest x-ray
Acyclovir 500 mg /m2 or 10-15 mg/kg every 8 hours

INFLUENZA INFLUENZA INFLUENZA


RNA virus MANAGEMENT: CATEGORY C DRUGS
Influenza A and B 1. Neuraminidase Inhibitors
Fever, dry cough and systemic infection a. Oseltamivir 75 mg BID x 5 days within 48 hours
DIAGNOSIS:
of symptoms
No firm evidence of malformation
Viral antigen rapid detection Assay b. Zenamivir (Relenza)
Rare transplacental passage
POLYMERASE CHAIN REACTION( PCR) c. Peramivir
Reports of stillbirth, preterm delivery and
(MOST SENSITIVE) 2. Adamantanes
abortion in severe cases
Rapid Influenza Diagnostic Test a. Amantadine
Immunofluorescence b. Rimantidine
Viral Culture

Fever, Coryza, Conjunctivitis, RUBEOLA (MEASLES)


INFLUENZA Cough RUBEOLA (MEASLES)
Erythematous maculopapular rash
VACCINATION Prevention: Exposure on Non Immune
(Face, neck, back, trunk,
Inactivated vaccine Pregnant
extremities)
IV Immunoglobulin 400 mg /kg within 6 days
RECOMMENDED FOR pregnant during Koplik spots (small white lesion
with erythema in oral cavity VACCINATION CONTRAINDICATED
influenza season DURING PREGNANCY but can be given
Dx: Serology,-RT-PCR
during post partum breastfeeding
Treatment: Supportive

Complications during pregnancy:


Abortion, preterm delivery, low
birth weight
RUBELLA ( GERMAN MEASLES) RUBELLA ( GERMAN MEASLES) CONGENITAL RUBELLA SYNDROME
Cataracts and congenital glaucoma
Fetal effects:
Patent ductus arteriosus and peripheral
Clinical S/Sx: Abortion
and severe congenital pulmonary artery stenosis
mild, febrile illness w/ a generalized maculopapular malformations Sensorineural deafnessthe most common
rash 1st 12 wks: 80% single defect
arthralgias
13-14 wks: 54% Central nervous system defects (Microcephaly,
arthritis
>14 wks: 25% mental retardation)
Lymphadenopathy
Pigmentary retinopathy
Suboccipital

Postauricular
Purpura
Cervical Hepatosplenomegaly /Jaundice
Conjunctivitis radioluscent bone densities

RUBELLA ( GERMAN MEASLES) RUBELLA ( GERMAN MEASLES) GROUP B STREPTOCOCCUS


Management:
Symptomatic Streptococcus agalactiae
Diagnosis: Droplet precautions Colonize the vagina and rectum
Clinical Characteristics:
Asymptomatic to sepsis
ELISA Adverse pregnancy outcome:

IgM acute/recent infection Prevention: 1.

2.
preterm labor
prematurely ruptured membranes
4-5 days after onset of rash up to 6 weeks
MMR vaccine (Not given during pregnancy) 3. clinical and subclinical chorioamnionitis
bacteriuria
IgG post infection (cant differentiate past
4.

Pyelonephritis
Pregnancy is avoided 1 month after
5.
immunity and recent disease) 6. postpartum endometritis
1-2 weeks after onset of rash vaccination 7. Postpartum maternal osteomyelitis and mastitis

High Avidity IgG antibodies indicate an infection

at least 2 months in the past

GROUP B STREPTOCOCCUS
GROUP B STREPTOCOCCUS ANTIBIOTIC REGIMEN
PATHOGENESIS & CLINICAL
TRANSMISSION MANIFESTATION
CAUSATIVE AGENT: T. PALLIDUM
Primary Syphilis
Painless chancre
ABRASION on the vagina (Portal of with raised red firm
Entry) & disseminate through lymphatic border and smooth
channels base, non
suppurative
SEXUALLY TRANSMITTED Incubation period 3-90 days (ave: 3 weeks) lymphodenopathy
DISEASES Fetal transmission: Transplacental, Resolves in
Willams 24th Edition contact through lesions of delivery 2-8weeks even if
Elizabeth Ahyong-Reyes, FPOGS untreated

CLINICAL CLINICAL CLINICAL


MANIFESTATION MANIFESTATION MANIFESTATION

Secondary Syphilis Secondary Syphilis Latent Syphilis


Develops 4-10 weeks
after chancre
Fever, malaise, anorexia, headache,
Diffuse macular rash, myalgias, arthralgias (+) serological testing with no clinical
palmar target like Rarely: aseptic meningitis, hepatitis, manifestation
lesions, patchy
alopecia, mucous
nephropathy, ocular changes, anterior Early latent within 12 weeks
patches uveitis, periostitis
Late latent - >12 months
Condyloma lata
(perineum)- flesh
colored papules &
nodules.

CLINICAL
MANIFESTATION CONGENITAL INFECTION NEWBORN
Hepatic Abnormalities
Tertiary or Late Syphilis Jaundice,purpuric skin
lesion
Slowly progressive disease affecting any
Anemia Lymphadenopathy
organ but rarely seen in reproductive age
woman Rhinitis
Thrombocytopenia Pneumonia

Myocarditis
Ascites & Hydrops Nephropathy

Long bone involvement


STILLBIRTH
PLACENTA DIAGNOSIS PERINATAL DIAGNOSIS
Definitive Diagnostic: Dark field Examination 1. Ultrasound: hydrops, ascites,
Large and pale & Direct Immunoflurescent Antibody hepatomegaly and placental thickening,
Screening Test: hydramnios
Chorionic villi: lose
arborization, thicker 1. Non Treponemal
and clubbed VDRL, RPR
2. Doppler Velocimetry elevated in
2. Treponemal - Specific
Diminish blood middle cerebral artery
FTA ABS, MHA TP, TP PA
vessels
(Trep. Pallidum Particle Agglutination)
3. PCR of AF / Darkfield Illumination of
AF

TREATMENT PENICILLIN ALLERGY GONORRHEA


History
of Allergy oral stepwise penicillin
dose challenge or skin testing
Penicillin Desensitization
Marker of concomitant chlamydial
JARISCH Herxheimer Reaction uterine
infection
contraction, FHR deceleration Limited to lower genital tract

COMPLICATIONS: septic abortion,


preterm delivery, PPROM,
chorioamnionitis, Postpartum infection

GONORRHEA TREATMENT GONORRHEA


1. SCREENING
Culture NEWBORN: Erythromycin, Tetracycline,
NAATC (Nucleic acid Amplification Test) Silver Nitrate
CHLAMYDIA SCREENING & TREATMENT HERPES SIMPLEX VIRUS
Most common infectious disease Screening

Mostly asymptomatic or urethral Culture (NAAT)


syndrome, urethritis or Bartholins gland Treatment

infection, Mucopurulent cervicitis


Perinatal Transmission Conjunctivitis/
Pneumonia

HSV 2

HERPES SIMPLEX VIRUS CLINICAL MANIFESTATION CLINICAL MANIFESTATION


1. First Episode Primary Infection 2. First Episode Non-primary Infection
CAUSATIVE AGENT: HSV is isolated with other HSV type
HSV 1 non genital Papule, itching and tingling vesicular antibody
HSV 2 Genital and painful Milder signs and symptoms

Lymphadenopathy

NEONATAL TRANSMISSION Transient Influenza like symptoms 3. Reactivation


1. Intrauterine (5%) Viral shedding of the HSV residing in the

2. Peripartum (85%) nerve ganglia


3. Postnatal (10%)

NEWBORN INFECTION DIAGNOSIS MANAGEMENT


Skin, eye, mouth lesions (40%) 1. Virological Test Acyclovir,Famcyclovir, Valacyclovir
CNS disease with encephalitis (30%) Cell culture & PCR (Preferred Test) (decrease duration of symptoms and viral
Disseminated disease of multiple organ * negative test does not exclude Infection
shedding)
involvement (32%) Oral Analgesic

Not associated with abortion and stillbirth Topical Anesthetic


2. Type Specific Serological Test
Indwelling catheter for urinary retention
ELISA or Blot style test for HSV
glycoproteins
PERIPARTUM SHEDDING HUMAN PAPILLOMA VIRUS
MANAGEMENT PROPHYLAXIS INFECTION
40 types infect genital tract
CS is recommended for women with active Type 16 & 18 oncogenic and high risk to
genital lesions or prodromal symptoms dysplasia
CS is not recommended for active lesions in non Type 6 & 11 mucocutaneous external genital
genital area wart
No evidence that external lesions causes
ascending infection in PPROM
Increase
in number to size during
pregnancy
Breastfeeding is allowed as long as there no
breast lesions

TREATMENT TREATMENT VAGINITIS


VACCINATION BACTERIAL VAGINOSIS
80-90%trichloracetic acid topically once a week Not an infection; fishy smelling discharge
Quadrivalent-Gardasil
Cryotherapy, laser or surgical excision Decrease lactobacilli and increase anaerobes
Bivalent-Cervarix (Gardnerella, Mobilincus & Bacteroides)
Contraindicated: podophyllin, podofilox,
Risk factors: drinking, multiple partners, young
imiquimod, interferon, sinecathechines
age, smoking, black race, and vitamin D
Laryngeal papillomatosis is rare. CS is no longer NOT given during pregnancy but can be deficiency
indicated to decrease transmission risk given to breastfeeding women. Complications: preterm birth, miscarriage low
birth weight, neonatal morbidity
Management: Metronidazole 500mg BID x 7days
or 250mg q8hrs x 7 days; Clindamycin 300mg
BID x 7days.

HUMAN
IMMUNODEFICIENCY WHAT AFFECT RAPIDITY OF CLINICAL SIGNS OF
VIRUS PROGRESSION: PROGRESSION
Causative agents: RNA retrovirus HIV 1 (most 1. Route of infection Generalized lymphadenopathy
common) & HIV 2
2. Pathogenicity of infecting viral strain Oralhairy leuplakia
Method of transmission
Blood or blood contaminated products
3. Initial viral inoculum Apthous ulcers
Perinatal 4. Immunological status of the host
Thrombocytopenia

breastfeeding
Primary method of Dx: HIV I viral load
MATERNAL & PERINATAL MANAGEMENT DURING
DIAGNOSTICS TRANSMISSION PREGNANCY
SCREENING: enzyme linked Vertical transmission serum creatinine, hemogram, and bacteriuria screening
immunoassay More common in prolonged membrane rupture Plasma HIV RNA quantificationviral load, CD4+ T-
lymphocyte count, and antiretroviral resistance testing
Delivery
Serum hepatic aminotransferase levels
CONFIRMATORY: westernblot or Breastfeeding HSV-1 and -2, cytomegalovirus, toxoplasmosis, and
immunoflourescence asssay hepatitis C serology screening
chest radiograph
Tuberculosis skin testingpurified protein derivative
(PPD)
Pneumococcal, hepatitis B, hepatitis A, Tdap, and
influenza vaccines
Sonographic evaluation

PREFERRED CLASSES OF
ANTIRETROVIRAL DRUGS DURING
LABORATORY COMPLICATIONS OF HIV
PREGNANCY ASSESSMENT INFECTION
1. Lamivudine CD4 T-lymphocyte count 1. Pneumocystic jiroveci pneumonia
2. Zidovudine HIV RNA viral load sulfamethoxazole trimetoprim or Dapsone
3. Ritonavir CBC

4. Atazanavir Liver function test 2. Opportunistic infections


5. Lopinavir/ ritonavir Glucose testing at 24-48 weeks Toxoplasmosis, herpes, mycobacteria, candida
6. Nevirapine
3. IUGR, preeclampsia, PPROM

PRENATAL HIV PRENATAL HIV


TRANSMISSION TRANSMISSION
Highly active aniretroviral therapy or CS has been recommended to decrease
HAART reduced perinatal HIV HIV prenatal transmission for patients
HEMATOLOGICAL DISORDERS OF
transmission with 1000 copies/ml of HIV RNA load at PREGNANCY
Avoid artificial rupture of membranes, 38 weeks
invasive fetal monitoring IV Zidovudine as loading dose prior to CS

Avoid forceps or vacuum extraction followed by 2 more hours of continuous


Methergine and other ergot adversely
maintainance therapy
interact with antiretroviral drugs causing
vasoconstriction
ANEMIA IRON-DEFICIENCY ANEMIA IRON-DEFICIENCY ANEMIA

hemoglobin concentration less than 12 g/dL in ETIOLOGIES: EFFECTS ON THE MOTHER:


Poor nutritional status increase maternal morbidity
nonpregnant women and less than 10 g/dL
deficiency in serum iron worsens the anemia
during pregnancy or the puerperium
consequence of expansion of plasma volume without EFFECTS ON THE FETUS:
normal expansion of maternal hgb mass not affected
The Centers for Disease Control and DIAGNOSIS: MANNER OF DELIVERY:
Prevention (1990) defined anemia as less erythrocyte hypochromia and microcytosis obstetrical indication
low serum ferritin levels MANAGEMENT:
than 11 g/dL in the first and third trimesters
non stainable bone marrow iron ferrous sulfate, fumarate, or gluconate 200 mg daily of
and less than 10.5 g/dL in the second
elemental iron
trimester transfusion of pRBC if hgb <8mg/dl or if to undergo
surgery like CS

ANEMIA FROM ACUTE BLOOD LOSS IMMUNE THROMBOCYTOPENIC PURPURA IMMUNE THROMBOCYTOPENIC PURPURA
ETIOLOGIES: ETIOLOGY: EFFECTS ON THE FETUS:
Abortion consequence of antibodies directed against platelets thrombocytopenia
ectopic pregnancy inc risk of intracranial hemorrhage
hydatidiform mole DIAGNOSIS: MANNER OF DELIVERY:
placenta previa intrapartum fetal scalp platelet determinations once Ceserean section
the cervix was 2 cm dilated and the membranes MANAGEMENT:
MANAGEMENT: ruptured
prednisone 1-2mg/kg if w/ significant bleeding & plt
Whole blood transfusion to maintain perfusion to vital percutaneous umbilical cord blood sampling ct <30,000/ul
organs splenectomy
hemostasis EFFECTS ON THE MOTHER: immuno-suppressive drugs
iron therapy for at least 3 mos benign condition gammma globulin 400mg/kg IV for 5 days
hemorrhagic complication

most common malignancy of women of all age Diagnosis and treatment:


groups Any
suspicious breast mass found - ultrasound, fine-
needle aspiration, or biopsy.

Risk of Mammography for the fetus is negligible

CORE BIOPSY may be an ideal diagnostic technique


Effects of pregnancy on breast cancer:
and Fine needle aspiration for cytology is an
pregnancy doesn t exert much influence on the alternative technique
course of breast cancer

Survival is stage dependent


Diagnosis and treatment: Surgical Treatment:
Positive for malignancy- chest x-ray , NOT DELAYED DUE TO PREGNANCY
Successful lactation and breast feeding are possible
wide excision, modified radical mastectomy, or total after conservative surgery and radiation for breast
mastectomy with axilllary node dissection cancer
MRI and Ultrasound to assess liver involvement
Chemotherapy is recommended for node-positive
disease

Cervical Neoplasia
Endocervical curettage NO done due to risk of
Delay pregnancy for 2 to 3 years- most critical Colposcopic evaluation is easier to perform hemorrhage and membrane rupture
observation period
Biopsies are used to assess any suspicious lesions Loop electrosurgical excision procedure (LEEP) and
cone biopsy - to exclude invasive cancer.

CONIZATION not done due to hemorrhage, abortion,


and preterm labor.

Management Management
DEPENDS on the STAGE and AOG most woman are asymptomatic
DEPENDS on:
First half of pregnancy- immediate treatment most pelvic masses are detected by routine 1.gestational age
without regard to pregnancy prenatal examination during the 1st trimester 2.stage
3 histological type
Latter half of pregnancy- await fetal 4. grade of the tumor
maturity,deliver and manage accordingly
IF confined to one ovary require complete surgical
staging, as do tumors of low malignant potential
PRURITIC URTICARIAL PAPULES
AND PLAQUES OF PREGNANCY
PRURITUS GRAVIDARUM (PUPPP)
Mild variant of Intrahepatic Cholestasis of
pregnancy
CM: intense pruritus, patchy or
generalized on abdomen, thighs,
arms, and buttocks,
erythematous urticarial plaques
or papules
Histopath: lymphocytic
perivascular infiltrate, (-)
immunoflourescence
Effects on Pregnancy: No
DERMATOSES OF PREGNANCY adverse effects
CM: Intense generalized pruritus, excoriations
Treatment: antipruritics,
Effects on Pregnancy: PERINATAL MORBIDITY IS emollients, steroids
INCREASED
Treatment: antipruritics, cholestyramine, The MOST COMMON
ursodeoxycholic acid pruritic pregnancy-
specific dermatoses

PRURITIC FOLLICULITIS OF
PRURIGO OF PREGNANCY PREGNANCY (PFP)
CM: generalized; erythema with
CM: usually in forearms and marginal sterile pustules; mucus
trunk; 1-5 mm often excoriated membrane involvement;
itchy papules (+)systemic symptoms
Histopath: lymphocytic Histopath: microabscesses,
perivascular infiltrates, spongiform pustules of Kogoj;
parakeratosis acanthosis, (-) neutrophils
immunoflourescence Effects on Pregnancy:
Effect on pregnancy: maternal sepsis common
probably none Treatment: Antibiotics, oral
Treatment: antipruritics, steroids
Comments:
steroids Spongiform Pustule of Kogoj

Aka. IMPETIGO
HERPETIFORMIS

HERPES GESTATIONIS
QUESTIONS:
CM: severe pruritus; abdomen,
extremities, or generalized; 1.28 y/o G2P1 PU 25 weeks develop low grade 2.20y/o G1P0 PU 12 weeks has been exposed to a
urticarial papules and plaques,
erythema, vesicles, and bullae fever followed development of tender,vesicular relative with varicella infection 2 days ago.She
Histopath: edema; infiltrate of lesions along the dermatome at the subcostal mentioned that she did not have the disease
lymphocytes, histiocyte and area.What is the risk of the fetus in developing during childhood.How will you manage this
eosinophils; C3 and IgG deposition
at basement membrane-(+) the disease? patient?
immunoflourescence a.none a.reassurance
Effects on pregnancy: possibly
increased preterm birth,; transient b. 10% b.vaccination
neonatal lesions (5-10%)
c. 20% c.immunoglobulin
Treatment: antipruritics, steroids
d. 30% d.vaccination and immunoglobulin

Aka. Pemphigoid gestationis


3.34y/o G3P3 delivered to a live baby with 4.What will differentiate if the patient had a 5.30 y/o G5P3 PU 35 weeks has uterine
cataracts,glaucoma and sensorineural recent rubella infection? contractions.She mentioned that her last baby
deafness.She mentioned that she developed high a.Ig M died of sepsis after delivery.What is the
grade fever with postauricular lympadenopathy recommended antibiotic prophylaxis ?
b.Ig G
and generalized maculopapular rashes during a.amoxicillin
c. High avidity Ig M
the first trimester of pregnancy.What is the b.ampicillin
disease that she had during the first trimester? d.High avidity Ig G
c.penicillin G
a.Rubeola
d.clindamycin
b.Rubella

c.Varicella

d.PUPP

6.30y/o G3P2 PU 14 weeks,complains of painless 7.32 y/0 G2P1 PU 23 weeks complains of 8.36y/o G3P1 PU 39 weeks was admitted in early
chancre at the vulva.The chancre has red and yellowish vaginal discharge.On gram stain,gram labor.On examination,there are multiple painful
firm border.What is the most specific diagnostic negative intracellular diplococci were seen.What vesicular lesions noted on the vulva.What is the
test for the patient? is the management? management??
a.RPR a.Azithromycin plus clindamycin a.insert an internal monitoring device

b.TPHA b.ceftriaxone plus metronidazole b.ask the nurse to prepare the forceps

c.darkfield illumination c.cetriaxone plus azithromycin c.prepare patient for cesarean section

d.ELIZA d.cefuroxime plus clindamycin d.amnitomy and induce with oxytocin

9.21 y/o G1P0 PU 12 weeks complains of vulvar 10.35 y/o G3P2 PU 34 weeks complains of 11.31y/o G2P1 PU 38 weeks is positive for HIV
itchiness.On inspection,there are multiple small premature uterine contractions.On speculum infection with a viral load of 2000 copies/ml.What
warty outgrowths noted on the labia majora and exam,there is a moderate amount of grayish is the BEST management?
perineum.What is the BEST management? homogenous fishy odored discharge.Grams stain a.Do amniotomy in early labor
a.Podophylline done revealed a nugent score of 8.What is the
b.Deliver by forceps during the second stage
b.trichloracetic acid
management?
c.Monitor condition of fetus by scalp sampling
a.amoxicillin
c.laser d.Deliver by Cesarean section
b.clindamycin
d.imiquimod
c.metronidazole

d.cefuroxime
12.32y/o G2P1 PU 36 weeks has Immune 13.32y/o G1Po PU 20 weeks complains of 14.36y/o G4P3 PU 10 weeks complained of
thrombocytopenia.What is the fetal complication palpable breast mass.On examination,a 2x3 cm postcoital bleeding.An ulcerated lesion was noted
anticipated if this patient will undergo vaginal solid mass was noted on the right upper on the cervic at 3 oclock position which bleeds to
delivery? quadrant of the breast.What is the BEST touch.Biopsy revealed squamous cell
a.vertebral fracture management? carcinoma.The uterus is not
b.intracranial hemorrhage a.mammogram enlarged ,movable,both parametria are free and
pliable.What is the management ?
c.liver rupture b.fine needle aspiration
a.chemotherapy and wait for viability
d.splenic injury c.breast ultrasound
b.cone biopsy and wait for delivery
d.core biopsy
c.chemotherapy and radiotherapy after delivery

d.radical hysterectomy with bilateral lymph node


dissection

15.Which of the following will NOT determine


the management of ovarian Ca during
pregnancy?
a.age of patient

b.stage of disease

c.gestational age

d.grade of the tumor

Вам также может понравиться